PDA

نسخه کامل مشاهده نسخه کامل : اتاق ریاضیات(طرح سؤالات)



صفحه ها : 1 [2] 3 4 5 6 7 8 9 10 11 12 13 14 15 16 17 18 19 20

mofidy1
22-08-2006, 19:22
با سلام

دوست عزیز امرتان اطاعت شد. برایتان دستور maple را به همراه شکل آن آپلود کردم. ضمنا عرض کنم که شکل به هیچ عنوان جای اثبات را نمی گیرد. شکل این دو تابع برای تکمیل دید شهودی مساله ارائه شد.

بنده از همان ابتدا در اولین پست، یکی از برنامه های اتاق ریاضیات را آموزش maple اعلام کرده بودم. اگر دوست عزیزی پیدا شد و این زحمت را به عهده گرفت که منتی بر این حقیر گذاشته است و گرنه خود بنده با تمامی مشکلاتم این کار را شروع خواهم کرد. انشاءالله.

در ضمن فکر می کنم بهترین شخص برای حل مسائل پست 177 علی آقای گل خودمان باشد (ali_hp). علی آقا « یا علی مدد» منتظریم.



[ برای مشاهده لینک ، لطفا با نام کاربری خود وارد شوید یا ثبت نام کنید ]


موفق باشید.

abay
22-08-2006, 22:57
دوست عزیز امرتان اطاعت شد. برایتان دستور maple را به همراه شکل آن آپلود کردم.
سلام
آقای مفیدی کجا آپلودش کردید

ali_hp
23-08-2006, 03:22
باسلام

فایلی را که آقا یا خانم 136784 آپلود کرده و خواستار حل مسائل آن شده اند، ذیلاً مشاهده می فرمایید. منتظر حل آنها توسط دوستان عزیز هستیم.
موفق باشید.
================================================== =====


[ برای مشاهده لینک ، لطفا با نام کاربری خود وارد شوید یا ثبت نام کنید ]
سلام خوب توضيح نداده ام ولي هرجا كه مبهم بود بگوييد تا دوباره توضيح دهم.
3)جمله اول برابر 2- است وجمله دهم برابر 0 است براي اينكه مجموع ده جمله ماكزيمم شود جملات دوم تا دهم را برابر 0 بايد انتخاب كنيم و ماكزيمم مجموع ميشود 2-. براي اينكه مجموع ده جمله مينيمم شود بايدجملات دوم تا نهم را برابر 2- انتخاب كنيم و مي نيمم مجموع ميشود 18-.
4)
[ برای مشاهده لینک ، لطفا با نام کاربری خود وارد شوید یا ثبت نام کنید ]
پس دنباله مفروض نزولي است.
5) تغيير متناهي تا از جملات يك دنباله تغييري در وضعيت همگراياواگرا بودن ان بوجود نمي اورد.
اين دنباله همگرابه صفر است.براي n>10 با افزايش n كاهش مي يابد براي n<9 هم با افزايش n كاهش مي يابد. ولي جمله نهم از جمله دهم كوچكتر است پس يكنوا نيست.
6) دنبالهa(n)l : مقدار n+cosn همواره مثبت است و sinn هم براي نامتناهي n مثبت است وبراي نامتناهي n منفي
(با توجه با دايره مثلثاتي به سادگي مي توان ديد كه n بي نهايت در ربع اول ودوم مي افتد و بي نهايت بارهم در ربع سوم وچهارم) پس (سينوسn)تقسيم بر (n باضافه كسينوسn) براي بي نهايت n مثبت است و براي بي نهايت n منفي است پس جزصحيح ‍{ (سينوسn)تقسيم بر (n باضافه كسينوسn)} براي بي نهايت n بزرگتر مساوي 0 است و براي بي نهايت n كوچكتر مساوي 1- پس a(n)l نمي تواند همگرا باشد.
دنباله b(n)l: براي n هاي فرد عبارت داخل جز صحيح برابر است با(cosn-1) تقسيم بر n كه منفي است و براي n هاي زوج هم عبارت داخل جز صحيح برابر( cosn+1) تقسيم برn است كه مثبت است پس مثل a(n)l مي توان ثابت كرد كه
b(n)l هم همگرا نيست.
7) صورت سوال مبهم است ومشخص نيست كه چه چيزي يك ميليمتر افرايش طولي دارد.اما اگر مساله بخواهد
مفهوم فيزيكي درستي داشته باشد بايد منظور سوال اين باشد:
....در اثر حرارت تابستان هرواحدطول يك ميليمتر افزايش طولي دارد....(واحد طول متر است!)
يعني اگر ميله اي به طول يك متر از جنس فلز سازنده كانتينر داشته باشيم بر اثر حرارت طولش مي شود 001/1
متر. اگر ميله اي به طول x از اين فلز داشته باشيم طولش مي شود x+0/001x .حالت كلي تر مساله راحل مي كنيم
فرض كنيد مكعب مستطيلي به ابعاد a,b.c داشته باشيم.و فلز سازنده اش هم طوري است كه ميله به طول x از جنس ان فلز بر اثر حرارت تابستان طولش مي شود x+xt (يا به ازاي هر واحد طول t متر افزايش طول داريم)
افزايش حجم كانتينر چقدر است؟
حجم اوليه:v=abc و حجم ثانويه را w فرض كنيد.
[ برای مشاهده لینک ، لطفا با نام کاربری خود وارد شوید یا ثبت نام کنید ]
چون معمولا t عدد كوچكي است از جملات داراي t^2,t^3 به دليل ناچيز بودن صرف نظر شده است.
به سادگي مي توان ديد كه ابعاد اوليه كانتينر عبارتند از 3و5/4و5/7 پس در اينجا حجم اوليه v=(3)(7/5)(4/5)=101/25
وt=0/001
پس:w-v=3(0/001)(101/25)=0/30375 (البته بر حسب متر اما گزينه ها برحسب ميلي متر هستند وگزينه دو ضحيح است)

mofidy1
23-08-2006, 11:04
با سلام

علی آقا از زحماتتان بسیار ممنونم. فقط با عرض معذرت باید نکته ای را خدمتتان عرض کنم. بسیاری از فرمولهای آپلود شده به وسیله شما بیش از حد طولانی است و این باعث می شود که در مرورگرهای IE و firefox (که فعلا پر استفاده ترین مروگرها هستند) مشکلات زیادی برای خواندن متن ایجاد شود. حتی در مرورگر firefox بعضی از قسمتهای متن اصلا خوانده نمی شوند و این معمولا باعث زدگی مطالعه کننده می شود. اگر برایتان امکان دارد فرمولهای طولانی را تصحیح کنید، یا اینکه از این بعد دقت لازم را مبذول فرمایید.

متشکرم.

ali_hp
23-08-2006, 11:21
سلام
ایا من باید این فرمولها را به صورت چند عکس قرار دهم ویا راه ساده تری نیز برای این کار هست؟

mofidy1
23-08-2006, 11:34
سلام
ایا من باید این فرمولها را به صورت چند عکس قرار دهم ویا راه ساده تری نیز برای این کار هست؟

هنگام نوشتن به زبان tex سطرهای طولانی را با تایپ علامت \\ در وسط دستورات (البته در جای مناسب) به دو سطر یا سه سطر یا ... تبدیل کنید.

موفق باشید.

abay
23-08-2006, 13:55
باسلام خدمت دوستان
آقای ali-hp به خاطر اینکه زحمت کشیدی وبه سوالات جواب دادی تشکر میکنم البته شما می تونی وقتی که یه قسمتهایی از فرمولها را نوشتی ENTERرا بزنی وبقیش زیرش بنویسی تا بجای که طولش بیشتر عرضش بیشتر بشه که در این صورت قابل خوندن میشه واینم میتونم خدمت آقای مفیدی عرض کنم که با مرورگر opera هم میشه این نوشته ها راکامل دید هم عکسهایی که با مرورگرهای دیگر دیده نمیشوند رو دید

abay
23-08-2006, 14:07
با سلام مجدد
یادم رفت تو پست قبلی بگم آقای مفیدی من در مورد زبان tex چیزای زیادی نمیدونم ودر حد دستوراتی که شما در وبلاگ خود قرار داده اید ونمی دونم چه جوری باید باهاش کرد نرم افزار لازم داره یا نه اگه میشه یه توضیح یا آموزشی بدهید البته می دانم که به گفته ی خودتان درگیر مسایل کاری هستید ویه چیز دیگه اینکه یه سایتی معرفی کرده بودید به نام mathlink آیا عضویت در این سایت رایگانه واگر اینطوره شما خودتون ثبت نام کردید یا نه

mofidy1
23-08-2006, 16:42
با سلام مجدد
یادم رفت تو پست قبلی بگم آقای مفیدی من در مورد زبان tex چیزای زیادی نمیدونم ودر حد دستوراتی که شما در وبلاگ خود قرار داده اید ونمی دونم چه جوری باید باهاش کرد نرم افزار لازم داره یا نه اگه میشه یه توضیح یا آموزشی بدهید البته می دانم که به گفته ی خودتان درگیر مسایل کاری هستید ویه چیز دیگه اینکه یه سایتی معرفی کرده بودید به نام mathlink آیا عضویت در این سایت رایگانه واگر اینطوره شما خودتون ثبت نام کردید یا نه

با سلام

دوست عزیز قبلا در موضوع زبان tex مفصلا صحبت کرده ایم. شما نرم افزاری که در پست 199معرفی کرده ایم دانلود کنید و به وسیله آن بدون استفاده از زبان tex فرمول مورد نظر را بسازید. در این پست طریقه تبدیل این فرمولها را به زبان tex توضیح داده ام. با مقایسه فرمولی که ساخته اید و زبان tex تبدیل شده، کم کم به زبان tex تسلط پیدا خواهید کرد.

در ضمن نرم افزار بسیار معروف و پیشرفته WinEdt نیز هست که مخصوص نوشتن مقالات تخصصی ریاضی است. اگر مایل بودید از لینک زیر آنرا دانلود کنید و پس از تسلط نسبی به زبان tex از آن برای نوشتن مقالات ریاضی استفاده کنید.

[ برای مشاهده لینک ، لطفا با نام کاربری خود وارد شوید یا ثبت نام کنید ] (حجم فایل حدود 6 مگ)

در مورد سوال دوم عرض کنم که بله، عضویت در این سایت رایگان است و بنده ماههاست که عضو این سایت هستم. فکر می کنم یکی از بهترین سایتهای حل مساله باشد.

ضمنا آیا شکل دوم مربوط به maple را که برایتان آپلود کرده بودم دیدید؟

موفق باشید.

abay
23-08-2006, 18:45
سلام
بله تصاویر آپلود شده را دیدم برای همه چی ممنون

ali_hp
23-08-2006, 23:31
هنگام نوشتن به زبان tex سطرهای طولانی را با تایپ علامت \\ در وسط دستورات (البته در جای مناسب) به دو سطر یا سه سطر یا ... تبدیل کنید.

موفق باشید.
سلام
در پيوندي كه در پست 155 معرفي كرده ايد اين دستور كار نمي كند!
در پيوندي كه در پست 28 معرفي كرده ايد اين دستور كار مي كند ولي من با ----- شكن به انجا رفتم واگر از انجا استفاده كنم پس از مدتي عكسها را ----- مي كنند.

mofidy1
24-08-2006, 00:41
با سلام

علی آقا این دستور مشکلی ندارد و خود بنده قبلا در همان جا این کار را انجام داده ام. منتهی احتیاج به کمی تجربه دارد. بهترین پیشنهاد بنده به شما این است که ابتدا یک فضای شخصی مناسب در گوگل، یاهو یا هر جای دیگر برای خود دست و پا کنید (علی آقا این کار را حتما انجام دهید چون برای کارهای علمی، بعدها به شدت به آن نیاز خواهید داشت)؛ سپس با نرم افزاری که در پست 141 ([ برای مشاهده لینک ، لطفا با نام کاربری خود وارد شوید یا ثبت نام کنید ]) آمده فرمولها را بسازید. در این نرم افزار و در قسمت file گزینه save as را کلیک و قسمت gif را در بخش save as type انتخاب کنید و پس از انتخاب دایرکتوری دلخواهتان و نام فایل، save را کلیک کنید. حال یک فرمول در قالب تصویر با حجم بسیار کم خواهید داشت. این تصویر را در فضای شخصی یا هر فضای مناسب دیگر آپلود کنید و آدرس تصویر آپلود شده را در صفحات وب برای استفاده دیگران قرار دهید.

موفق باشید.

abay
24-08-2006, 15:00
با سلام
من یه سوالی درمورد خط در فضا داشتم وآن اینکه می دانیم زاویه بین دو خط در فضا همان زاویه بین بردارهای هادی خط است ویا به عبارتی داریم زاویه ی بین دو خط زاویه ی حاده ویا قایمه دو خط است ومن می خواستم بدونم که چرا زاویه ی منفرجه بین دوخط زاویه بین دو خط محسوب نمی شود و در واقع وقتی زاویه بین دو خط خواسته می شود منظور همان زاویه ی حاده یا قائمه دو خط است مثلا"در دوخط متقاطع زیر داریم
[ برای مشاهده لینک ، لطفا با نام کاربری خود وارد شوید یا ثبت نام کنید ]
که با توجه به تعریف که زاویه ی بین دو خط همان زاویه ی بین بردارهای هادی است داریم:
[ برای مشاهده لینک ، لطفا با نام کاربری خود وارد شوید یا ثبت نام کنید ]
درحالیکه زاویه بین دو بردارمکمل این زاویه محسوب میشود [ برای مشاهده لینک ، لطفا با نام کاربری خود وارد شوید یا ثبت نام کنید ]
لطفا" راهنمایی کنید
اگر آقای مفیدی برایتان میسر است نمودار دو خط بالا وهمچنین معادله دو خطی که در پایین داده ام را به وسیله نرم افزار میپل رسم کنید البته این معادله ی پایین به دو خط متنافر است اگه با این نرم افزار عمود مشترک این دو خط قابل رسم باشد این راهم لطف کنید
[ برای مشاهده لینک ، لطفا با نام کاربری خود وارد شوید یا ثبت نام کنید ]

mofidy1
24-08-2006, 18:34
با سلام

دستورات مربوط به خط اول را برایتان آپلود کردم. بی زحمت خودتان با تقلید از این دستورات (و قضایای هندسه 2 سوم ریاضی) بقیه را رسم کنید.

موفق باشید.


[ برای مشاهده لینک ، لطفا با نام کاربری خود وارد شوید یا ثبت نام کنید ]

mofidy1
25-08-2006, 12:22
با سلام

در پست 164 آقای منبتی مساله ی زیبایی را مطرح کردند که وقتی به حل آن مشغول شدم زیبایی آن برایم دو چندان شد. متن مساله بسیار ساده و به ظاهر پیش پا افتاده است، اما فکر نمی کنم از روشهای ساده و معمولی قابل حل باشد. اگر دوستان راه حلی برای آن هر چند طولانی پیدا کردند در اینجا مطرح فرمایند. متشکرم.

مساله:

ثابت کنید تابعی با شرایط زیر وجود ندارد:


[ برای مشاهده لینک ، لطفا با نام کاربری خود وارد شوید یا ثبت نام کنید ]

موفق باشید.

ارسال متن: شنبه 28 مرداد 1385

با سلام

به برهان خلف عمل می کنیم. فرض کنید چنین تابعی وجود داشته باشد. تعریف کنید:


[ برای مشاهده لینک ، لطفا با نام کاربری خود وارد شوید یا ثبت نام کنید ]

توجه کنید که


[ برای مشاهده لینک ، لطفا با نام کاربری خود وارد شوید یا ثبت نام کنید ]

نقاط 1- و 2 را نقاط ثابت تابع g می نامیم. اگر نقطه ای مانند x نقطه ثابت g باشد آنگاه (f(x نیز نقطه ثابت g خواهد بود. لذا با توجه به تعریف g می توان نوشت:


[ برای مشاهده لینک ، لطفا با نام کاربری خود وارد شوید یا ثبت نام کنید ]

حال g را با خودش ترکیب کنید و به یک چندجمله ای درجه 4 به نام h برسید. این چندجمله ای را با x مساوی قرار دهید و پس از حل معادله به دست آمده، نقاط ثابت h را به دست آورید. مجموعه نقاط ثابت h را F بنامید. لذا:


[ برای مشاهده لینک ، لطفا با نام کاربری خود وارد شوید یا ثبت نام کنید ]

همانند قبل می توان استدلال کرد که مجموعه زیر با F برابر است:


[ برای مشاهده لینک ، لطفا با نام کاربری خود وارد شوید یا ثبت نام کنید ]

سومین عنصر F را a و چهارمین عنصر آنرا b بنامید و فرض کنید x ، عنصر a یا b باشد و (f(x را در نظر بگیرید. اگر


[ برای مشاهده لینک ، لطفا با نام کاربری خود وارد شوید یا ثبت نام کنید ]

که تناقض است. بنابر این


[ برای مشاهده لینک ، لطفا با نام کاربری خود وارد شوید یا ثبت نام کنید ]

به راحتی می توان دید که a و b نمی توانند نقاط ثابت f باشند لذا


[ برای مشاهده لینک ، لطفا با نام کاربری خود وارد شوید یا ثبت نام کنید ]

که تناقض اصلی است. (توجه کنید که با توجه به ایده های به کار رفته در حل این مساله می توان مساله را تعمیم داد.)

موفق باشید.

ارسال متن: جمعه 3 شهریور 1385

ali1234
26-08-2006, 09:09
با سلام وخسته نباشيد خدمت همه عزيزان
يه سوال داشتم :
[ برای مشاهده لینک ، لطفا با نام کاربری خود وارد شوید یا ثبت نام کنید ]
با تشكر.


سلام آقاي مفيدي
ما هنوز چشم انتظاريم.

mofidy1
26-08-2006, 14:07
با سلام

علی آقا به روی چشم. در اولین فرصت به این مساله خواهم پرداخت. راستش را بخواهید نمی خواهم در این اتاق متکلم وحده باشم. بیشتر مایلم که دوستان مساله را حل کنند. متاسفانه فرصت بنده هم بسیار محدود است و قبل از اتمام تابستان باید به کارهای زیادی برسم.

موفق باشید.

mofidy1
26-08-2006, 14:17
با سلام

مساله:

کوچکترین عدد طبیعی n را بیابید که اگر آخرین رقم سمت راست آنرا به سمت چپ انتقال دهیم، عدد حاصل «سه دوم» عدد n شود.

موفق باشید.

ارسال متن: شنبه 4 شهریور 1385

eh_mn
26-08-2006, 20:28
با سلام وخسته نباشيد خدمت همه عزيزان
يه سوال داشتم :
[ برای مشاهده لینک ، لطفا با نام کاربری خود وارد شوید یا ثبت نام کنید ]
با تشكر.

با سلام
در مورد این سوال دو سوال داشتم
1. آیا در مورد [ برای مشاهده لینک ، لطفا با نام کاربری خود وارد شوید یا ثبت نام کنید ] مطمئن هستید ؟
2. منظور از [ برای مشاهده لینک ، لطفا با نام کاربری خود وارد شوید یا ثبت نام کنید ] چيست ؟
(سمت راست تساوي يك بردار است و سمت چپ آن يك عدد)

موفق باشید

abay
26-08-2006, 23:53
با سلام

دستورات مربوط به خط اول را برایتان آپلود کردم. بی زحمت خودتان با تقلید از این دستورات (و قضایای هندسه 2 سوم ریاضی) بقیه را رسم کنید.

موفق باشید.


[ برای مشاهده لینک ، لطفا با نام کاربری خود وارد شوید یا ثبت نام کنید ]
سلام
آقای مفیدی از شما تشکر میکنم بخاطر اینکه به سوالات من جواب دادید اما به سوال اصلی بنده که مربو ط به زاویه ها ی دو خط بود جواب ندادید ددر ضمن نموداری که برای معادله ای که من نوشتم وشما رسم کرده اید من دستورش را در میپل نوشتم اما اخطار میده لطفا" را هنمایی کنید

ali1234
27-08-2006, 09:20
با سلام
در مورد این سوال دو سوال داشتم
1. آیا در مورد [ برای مشاهده لینک ، لطفا با نام کاربری خود وارد شوید یا ثبت نام کنید ] مطمئن هستید ؟
2. منظور از [ برای مشاهده لینک ، لطفا با نام کاربری خود وارد شوید یا ثبت نام کنید ] چيست ؟
(سمت راست تساوي يك بردار است و سمت چپ آن يك عدد)

موفق باشید

با سلام
در مورد سوال اول كه مطمئن هستم.اما در مورد سوال دوم سمت راست تساوي يك ماتريس ستوني و سمت چپ يك ماتريس سطري است وضرب هم ؛مسلما ضرب ماتريسي است.

ali1234
28-08-2006, 09:45
با سلام خدمت همه دوستان عزيز
آقاي مفيدي
در مورد مسئله هفته چهاردهم
آيا منظور شما از انتقال رقم سمت راست آن به سمت چپ اين است كه مثلا اگر عدد طبيعي ما 2485 باشد آنگاه به عدد 5248 تبديل شود؟
اگر اينطور است كه من توسط maple يه procedure نوشتم كه اين كار رو براي اعداد 10 تا 100000 انجام مي دهد ولي هيچ عددي پيدا نكرد.؟(اگر جايي از procedure اشكال دارد اطلاع دهيد)
همچنين در مورد نحوه حل مساله يه راهنمايي كنيد چون تا جايي كه اطلاع دارم راهي جز آزمون وخطا كه معمولا توسط نوشتن يك برنامه كامپيوتري صورت مي گيرد براي حل اين جور مسا ئل وجود ندارد !!!

[ برای مشاهده لینک ، لطفا با نام کاربری خود وارد شوید یا ثبت نام کنید ]

Saeed-a-s
28-08-2006, 12:54
بسمه تعالي
با سلام.
شرط برقراري يك مثلث قائم الزاويه چيست. براي مثال چه شرط هايي بايد براي سه عدد برقرار باشد تا بتوانيم با آن سه عدد يك مثلث قائم الزاويه را بسازيم.
با تشكر

roonak2006
28-08-2006, 13:31
يكي از شرطها كه فكر مي كنم شرط لازم و كافي باشه اينه كه مجموع مربعات دو ضلع قائم برابر با مربع ضلع سوم (وتر) باشد.

ali_hp
28-08-2006, 13:39
سلام
دوست عزيز لطفا سوالات رياضي خود را در اتاق رياضيات ([ برای مشاهده لینک ، لطفا با نام کاربری خود وارد شوید یا ثبت نام کنید ]) مطرح كنيد.

Saeed-a-s
28-08-2006, 13:48
لطفا واضح تر بفرمائيد كه من با اين سه عدد چه كنم. خيلي متشكر
در ضمن همين سوال را در آنجا مطرح مي كنم.

Saeed-a-s
28-08-2006, 13:57
-----------------------------------------------------------------------------
بسمه تعالي

رياضيدانهاي عزيز:
با سلام.
من سه عدد دارم مي خواهم ببينم چه شرطهايي بايد برقرار باشد تا من بتوانم با اين سه عدد يك مثلث قائم الزاويه بسازم و اين را هم براي برنامه نويسي مي خواهم - لطفا جواب را به صورت ساده بيان كنيد.
با تشكر

ali_hp
28-08-2006, 14:03
سلام
عدد بزرگتر را c بناميد و دو عدد ديگر را a,b.براي اينكه با اين سه عدد بتوان مثلث قايم الزاويه ساخت بايد هر سه مثبت باشندو همچنين بايد رابطه زير برقرار باشد:
توان دو a +توان دو b =توان دوc

Saeed-a-s
28-08-2006, 14:10
علي جان بابت جواب متشكرم
اما متاسفانه در آن لينكي را كه رابطه موردنياز در آن هست چيزي پيدا نكردم اگه ممكنه آن رابطه را در همين جا بذار.
با تشكر مجدد.

ali_hp
28-08-2006, 14:18
مي بخشيد لينكي كه گزاشتم اشتباه بود ولي پست ويرايش كردم ببينيد متوجه مي شويد؟

Saeed-a-s
28-08-2006, 14:20
جوابمو گرفتم - لطف كرديد - متشكر
اجركم عند الله تعالي

mofidy1
28-08-2006, 19:18
با سلام خدمت همه دوستان عزيز
آقاي مفيدي
در مورد مسئله هفته چهاردهم
آيا منظور شما از انتقال رقم سمت راست آن به سمت چپ اين است كه مثلا اگر عدد طبيعي ما 2485 باشد آنگاه به عدد 5248 تبديل شود؟
اگر اينطور است كه من توسط maple يه procedure نوشتم كه اين كار رو براي اعداد 10 تا 100000 انجام مي دهد ولي هيچ عددي پيدا نكرد.؟
همچنين در مورد نحوه حل مساله يه راهنمايي كنيد چون تا جايي كه اطلاع دارم راهي جز آزمون وخطا كه معمولا توسط نوشتن يك برنامه كامپيوتري صورت مي گيرد براي حل اين جور مسا ئل وجود ندارد !!!



با سلام

علی آقا از تلاشتان برای حل مساله به وسیله maple و نیز برنامه زیبایی که برای آن طراحی کرده اید، بسیار ممنونم. لازم است ذکر کنم که عدد مورد نظر از دویست هزار بیشتر است.

راهنمایی: عدد مورد نظر را m در نظر بگیرید. می خواهیم m سه دوم n باشد. m و n را در مبنای ده بسط دهید و از همنهشتی به پیمانه 4 و سپس 7 استفاده کنید.

موفق باشید.

rap.vartan
29-08-2006, 10:55
سلام
يه معما
خواهشا جوابشو زودتر بدين

يك مدرسه 1000 تا دانش آموز داره و هر دانش آموز يك كمد اختصاصي براي افتتاح مدرسه مدير براي تست كردن هوش دانش آموزان يك مسئله طرح مي كنه
همه ي بچه ها رو به صف مي كنه و در تمام كمد ها را باز مي گذارد وكليد هم روي در .
نفر اول را مي فرستد تا تمام در ها را ببندد و نفر دوم مضارب 2 را (2و4و6و...) تغيير حالت مي دهد (اگر باز باشد مي بندد و اگه بسته باشد باز ميكند) و اي آخر تا نفر 1000ام كه فقط حالت كمد 1000 رو تغيير ميده

و در آخر مدير هدايايي تهيه كرد
و از بچه ها سوال كرد كه چند در باز است ؟


همين ...

مرسي
ممنون

ali_hp
29-08-2006, 13:21
سلام
دقت كنيد كه درn ام دقيقا به تعداد مقسوم عليهايn تغيير وضعيت مي دهد.ودر ابتدا همه درها باز است. واضح است در نهايت در هايي باز مانده است كه به تعداد زوجي تغيير حالت داده باشد و يا به عبارت ديگر تعداد زوجي مقسوم عليه داشته باشند.پس بايد تعداد اعداد كوچكتر مساوي هزار را پيدا كنيم كه تعداد زوجي مقسوم عليه دارند.
قضيه: فقط وفقط اعداد طبيعي مربع كامل تعداد فردي مقسوم عليه دارند.(نتيجه:اعدادي كه مربع كامل نيستند تعداد زوجي مقسوم عليه دارند)
پس به تعداد اعداد كمتر مساوي هزار كه مربع كامل نيستند در باز خواهيم داشت.چون اعداد مربع كامل كوچكتر مساوي هزار 31 اي هستند(يك به توان دو و دو به توان دو و ....و سي يك به توان دو) پس تعداد اعدادي كه مربع كامل نيستندو ياتعداددرهاي باز مي شود 969.
اثبات قضيه:فرض كنيد p1,p2,...,pk عوامل اول عدد دلخواه n باشند.و در تجزيه n به عوامل اول با توانهاي
a1,a2,...ak ظاهر شده باشند مي دانيم تعداد مقسوم عليهاي n برابر است با:
a1+1)(a2+1)...(ak+1)l) پس n تعداد مقسوم عليهايش فرد است اگر وتنها اگرمقدار
a1+1)(a2+1)...(ak+1)l) فرد باشد كه به اين معني است كه تك تك عوامل اي حاصلضرب يعني a1+1,...ak+1
فرد باشند ومعادل اين است كه a1,...,ak زوج باشند كه اي هم معادل مربع كامل بودن n است.(يك عدد طبيعي مربع كامل است اگر وتنها اگر در تجزيه به عوامل اول ان عدد توان همه عوامل زوج باشد)

rap.vartan
29-08-2006, 14:22
واقعا ممنون

يك دنيا تشكر

ali1234
30-08-2006, 09:58
با سلام
برنامه اي رو كه براي حل مسئله هفته چهاردهم نوشته بودم بسط دادم به طوري كه اعداد طبيعي تا يك ميليون را كه داراي اين ويژگي باشند مشخص مي كند.كه فقط دوعدد در اين بازه داراي اين ويژگي هستند اولينش 285714 (جواب مسئله)وديگري 571428 .

eh_mn
30-08-2006, 11:42
با سلام وخسته نباشيد خدمت همه عزيزان
يه سوال داشتم :
[ برای مشاهده لینک ، لطفا با نام کاربری خود وارد شوید یا ثبت نام کنید ]
با تشكر.
با سلام
حل اين مسئله را در حالت n=2 بصورت زير است. در حالت كلي روش حل بهمين ترتيب است.
فرض كنيم [ برای مشاهده لینک ، لطفا با نام کاربری خود وارد شوید یا ثبت نام کنید ] . دو حالت در نظر مي گيريم:
1. حداقل يكي از x و y مثبت است. بدون اينكه ادامه حل دچار مشكل شود مي توانيم فرض كنيم x>0. در اين صورت

[ برای مشاهده لینک ، لطفا با نام کاربری خود وارد شوید یا ثبت نام کنید ]
از رابطه فوق يكبار برحسب x و بار ديگر برحسب y مشتق گيري مي كنيم. داريم

[ برای مشاهده لینک ، لطفا با نام کاربری خود وارد شوید یا ثبت نام کنید ]
و

[ برای مشاهده لینک ، لطفا با نام کاربری خود وارد شوید یا ثبت نام کنید ]
لذا

[ برای مشاهده لینک ، لطفا با نام کاربری خود وارد شوید یا ثبت نام کنید ]
2. x و y هر دو نامثبت اند. مشابه روند قسمت 1 عمل مي كنيم با تفاوت كه حل را با فاكتور گيري از x- آغاز مي كنيم.

موفق باشيد

mofidy1
30-08-2006, 11:47
با سلام
برنامه اي رو كه براي حل مسئله هفته چهاردهم نوشته بودم بسط دادم به طوري كه اعداد طبيعي تا يك ميليون را كه داراي اين ويژگي باشند مشخص مي كند.كه فقط دوعدد در اين بازه داراي اين ويژگي هستند اولينش 285714 (جواب مسئله)وديگري 571428 .

با سلام

احسنت. اما علی آقا مشخص است که این حل مساله نیست.

در ضمن علی آقا با maple چه مدتی است که کار می کنید؟ فکر می کنید توانایی شروع یک کلاس آموزش مقدماتی maple را در این اتاق دارید؟

منتظرم.

roonak2006
31-08-2006, 12:26
نگاه كنيد اگر اندازه هر كدوم از اضلاع مثلث را داشته باشين و ضلع شماره 1 و 2 اضلاع قائم باشند و شماره 3 وتر باشه , اونوقت هر كدوم از اينها را به توان 2 برسونيد. مي بينيد كه مجموع توان دوي ضلع 1 و 2 برابر با توان دوي ضلع 3 ميشه

ali1234
01-09-2006, 09:04
با سلام
در مورد سوال هفته چهاردهم در اينكه اين راه حل مسئله نيست . شكي نيست. البته من با توجه به راهنمايي هاي شما تلاش هايي براي حل مسئله كردم ولي به نتيجه اي نرسيدم .
در مورد ميپل هم بايد عرض كنم كه من آن را فقط به عنوان يه واحد درسي گذرا ندم و كار زيادي با آن انجام ندادم . و فكر نكنم بتوانم نياز هاي دوستان رو براورده كنم.البته يه لينك پيدا كردم كه در اون ميپل رو به صورت گام به گام واز سطح مقدماتي تا پيشرفته آموزش داده است وفكر كنم نيازهاي دوستان رو برآورده كند حده اقل مي تونم بگم خيلي بيشتر از اون چيزهايي كه من بلدم اونجا مي توانند بدست بياورند.

[ برای مشاهده لینک ، لطفا با نام کاربری خود وارد شوید یا ثبت نام کنید ]
موفق باشيد.

ali1234
01-09-2006, 09:52
با سلام
با تشكر از eh_mn عزيز .فقط يه جايي از اون متوجه نشدم .آنجايي كه با كادر قرمز مشخص كردم .(به نظر خودم بايد اونجا x باشد).

[ برای مشاهده لینک ، لطفا با نام کاربری خود وارد شوید یا ثبت نام کنید ]

mofidy1
01-09-2006, 14:08
با سلام

مساله:

کوچکترین عدد طبیعی n را بیابید که اگر آخرین رقم سمت راست آنرا به سمت چپ انتقال دهیم، عدد حاصل «سه دوم» عدد n شود.

موفق باشید.

ارسال متن: شنبه 4 شهریور 1385

با سلام

با نمایش اعداد طبیعی در مبنای 10 و با توجه به فرض مساله می نویسیم:


[ برای مشاهده لینک ، لطفا با نام کاربری خود وارد شوید یا ثبت نام کنید ]

فرض کنید:


[ برای مشاهده لینک ، لطفا با نام کاربری خود وارد شوید یا ثبت نام کنید ]

(قبل از ادامه حل مساله به این دو مثال توجه کنید: 31+2*2^10=231 و 2+568*10=5682)

حال می توان نوشت:


[ برای مشاهده لینک ، لطفا با نام کاربری خود وارد شوید یا ثبت نام کنید ]

کمترین مقدار k که در این رابطه صدق می کند عبارت است از 5 و کوچکترین مقدار n به وسیله کوچکترین مقدار k و a0 به دست می آید. حال با توجه به رابطه (1) :


[ برای مشاهده لینک ، لطفا با نام کاربری خود وارد شوید یا ثبت نام کنید ]

این مساله یکی از مسائل مسابقات ریاضی دانش آموزان کشور در سال 1366 بود (کتاب المپیاد ریاضی کشور تالیف دکتر عبادالله محمودیان).

موفق باشید.

ارسال متن: جمعه 10 شهریور 1385

mofidy1
02-09-2006, 12:13
با سلام

فرض کنید 1=x+y+z که y، x و z اعداد حقیقی نامنفی هستند. ثابت کنید:


[ برای مشاهده لینک ، لطفا با نام کاربری خود وارد شوید یا ثبت نام کنید ]

موفق باشید.

ارسال متن: شنبه 11 شهریور 1385

eh_mn
03-09-2006, 10:35
با سلام
با تشكر از eh_mn عزيز .فقط يه جايي از اون متوجه نشدم .آنجايي كه با كادر قرمز مشخص كردم .(به نظر خودم بايد اونجا x باشد).

[ برای مشاهده لینک ، لطفا با نام کاربری خود وارد شوید یا ثبت نام کنید ]

با سلام
فكر مي كنم همان y درست باشد.

قاعده زنجيري در توابع چند متغيره :
فرض كنيد [ برای مشاهده لینک ، لطفا با نام کاربری خود وارد شوید یا ثبت نام کنید ] و [ برای مشاهده لینک ، لطفا با نام کاربری خود وارد شوید یا ثبت نام کنید ] در اينصورت
[ برای مشاهده لینک ، لطفا با نام کاربری خود وارد شوید یا ثبت نام کنید ]
و
[ برای مشاهده لینک ، لطفا با نام کاربری خود وارد شوید یا ثبت نام کنید ]

موفق باشيد

ali1234
04-09-2006, 09:36
سلام eh_mn جان
مي خواستم u=u(x,y), v=v(x,y) , را در اين مسئله مشخص كنيد.i

eh_mn
04-09-2006, 12:45
سلام eh_mn جان
مي خواستم u=u(x,y), v=v(x,y) , را در اين مسئله مشخص كنيد.i

سلام و خسته نباشيد
فرض كنيم
[ برای مشاهده لینک ، لطفا با نام کاربری خود وارد شوید یا ثبت نام کنید ]
در اينصورت بنابه قاعده زنجيره اي داريم
[ برای مشاهده لینک ، لطفا با نام کاربری خود وارد شوید یا ثبت نام کنید ]
همچنين
[ برای مشاهده لینک ، لطفا با نام کاربری خود وارد شوید یا ثبت نام کنید ]

بنابراين نه حرف من نه حرف شما !! :blush: مشتق نسبي f نسبت به v درست است.

موفق باشيد

abay
04-09-2006, 20:28
با سلام

فرض کنید 1=x+y+z که y، x و z اعداد حقیقی نامنفی هستند. ثابت کنید:


[ برای مشاهده لینک ، لطفا با نام کاربری خود وارد شوید یا ثبت نام کنید ]

موفق باشید.

ارسال متن: شنبه 11 شهریور 1385
[ برای مشاهده لینک ، لطفا با نام کاربری خود وارد شوید یا ثبت نام کنید ]

ali1234
05-09-2006, 09:40
سلام و خسته نباشيد
فرض كنيم
[ برای مشاهده لینک ، لطفا با نام کاربری خود وارد شوید یا ثبت نام کنید ]
در اينصورت بنابه قاعده زنجيره اي داريم

بنابراين نه حرف من نه حرف شما !! :blush: مشتق نسبي f نسبت به v درست است.

موفق باشيد
با سلام
دستت دردنكنه
حلا كاملا دقيق شده
موفق باشي.

ali1234
05-09-2006, 12:12
سلام abay جان
با عرض خسته نباشيد
مي خواستم ببينم رابطه هاي زير رو چگونه نتيجه گرفته ايد؟!!

[ برای مشاهده لینک ، لطفا با نام کاربری خود وارد شوید یا ثبت نام کنید ]

abay
05-09-2006, 21:14
سلام abay جان
با عرض خسته نباشيد
مي خواستم ببينم رابطه هاي زير رو چگونه نتيجه گرفته ايد؟!!

[ برای مشاهده لینک ، لطفا با نام کاربری خود وارد شوید یا ثبت نام کنید ]
[ برای مشاهده لینک ، لطفا با نام کاربری خود وارد شوید یا ثبت نام کنید ]

manoxpwin
06-09-2006, 02:35
there is any formula to solve FACTORIAL (1*2*3*4*5*6*.....)
if there is pleas help me soory i cant write farsi why idont no

ali1234
06-09-2006, 11:54
[ برای مشاهده لینک ، لطفا با نام کاربری خود وارد شوید یا ثبت نام کنید ]

mofidy1
06-09-2006, 20:01
«مباحثی پیرامون روشهای درست مطالعه ریاضیات»

با سلام

یکی از دوستان خوب دانش آموز، در نامه ای خصوصی سوالی قریب به مضمون زیر را مطرح کردند:

«من دانش آموز سوم دبيرستان رشته ي رياضي هستم و در يكي از دبيرستانهاي شيراز درس ميخوانم.
آقاي مفيدي من رياضیات و فيزيكم خوبه و در سطح بالاست اما امسال ميخوام طوري باشم كه هم در تست و هم در تشريحي موفق ترين باشم چه در حسابان و ...
لطفا منو راهنمايي كنيد كه چه جوري بخونم و روزي چه قدر، چه درسهايي رو مطالعه كنم كه آمادگي براي هر گونه تست و تشريحي رو داشته باشم و از چه کتابهایی استفاده كنم»

بنده به عنوان یک معلم کوچک، این سوالات را بارها و بارها شنیده ام و متناسب با فرد سوال کننده به آن پاسخ داده ام. سوال این دوست عزیز، بهانه بسیار خوبی است که در این اتاق به طور مفصل به این پرسشها - که کاملا به حق و مورد نیاز بسیاری از دانش آموزان و حتی دانشجویان است - پاسخ دهیم و صد البته با این کار به یکی از اهداف اتاق ریاضیات نیز جامه عمل بپوشانیم. سعی می کنیم فقط در همین پست به اینگونه سوالات پاسخ دهیم و از دوستان دیگر نیز انتظار داریم که با نقد مطالبی که در این پست خدمتتان تقدیم می شود و یا با ارائه تجربیات خود در حد امکان به اینگونه سوالات پاسخ دهند.

«یا علی مدد»
-----------------------------------------------------------------

سوال بالا را به دو مرحله تقسیم می کنیم:

(الف) چگونه می توان کتب درسی ریاضی را به طور عمقی مطالعه کرد؟
(ب) چگونه می توان در تست زدن موفق شد؟ آیا واقعاً راه میانبری - همانگونه که بسیاری از موسسات کنکور ادعا می کنند - وجود دارد؟

به هر یک از دو سوال بالا به شیوه ترتیبی و البته به صورت کاملا خلاصه پاسخ می دهیم. دوستان عزیز در هر مورد ، اگر ابهامی دیدید بفرمایید تا درباره آن بیشتر بحث کنیم.

پاسخ سوال (الف):

1- برای خودتان برنامه هفتگی داشته باشید به گونه ای که اگر کسی از شما پرسید مثلاً روز دوشنبه ساعت 10 صبح یا پنجشنبه ساعت 5 بعد از ظهر قرار است چه کنید، برای آن پاسخ دقیقی داشته باشید. برنامه شما باید کاملا متعادل و به دور از هر گونه افراط و تفریط باشد. یک نوجوان دانش آموز و یا یک جوان دانشجو برای پیشرفت خود، غیر از فعالیتهای عمیق علمی متناسب با رشته خود، احتیاج به استراحت و خواب مناسب (حداقل 7 ساعت)، ورزش، دیدار دوستان و آشنایان، شرکت در فعالیتهای عبادی، اجتماعی، فرهنگی و سیاسی، دیدن برنامه های تلوزیونی، مطالعات غیر درسی مانند مطالعه روزنامه ها، مجلات، رمان و ... دارد. برنامه را به گونه ای طراحی کنید که اولا همه فعالیتهای لازم (حتی خواب و بیداری و غذا خوردن) شما را پوشش دهد و ثانیا شما را خسته نکند. توجه کنید که همه روشهای مطالعه که بعد از این توضیح خواهیم داد، باید تحت همین برنامه سازماندهی شود.

2- متن درس را مانند کسی بخوانید که می خواهد آنرا تدریس کند. حال ببینیم یک معلم خوب قبل از تدریس چه می کند: او با استفاده از تجربیات قبلی خود، ابتدا درس را کاملا و به طور عمیق مطالعه و سپس از مطالب آن خلاصه برداری می کند. به مطالب و تمرینات کتاب بسنده نمی کند و به وسیله کتب معتبر ، مطالب و مسائل جدید و جالبی به طرح درس خود می افزاید. گاهی هم برای اینکه بهتر و راحت تر تدریس کند، جداولی تهیه می کند و یا وسایلی با دست خود می سازد.

بنابر این «اگر می خواهید خوب بخوانید، همانند یک معلم بخوانید.» اگر برایتان امکان دارد درس را برای دیگری تدریس کنید و به او اجازه دهید از شما سوالاتی درباره همان درس بپرسد. اگر چنین امکانی برایتان نیست، بعد از مطالعه و خلاصه برداری، کتاب را کنار بگذارید و همانند یک معلم همان درس را برای خودتان تدریس کنید. دقت کنید که میزان مهارت شما در تدریس یک درس معمولا برابر است با میزان فهم مطالب آن درس توسط شما.

3- خودتان را به فکر کردن روی مساله های ریاضی عادت دهید. توجه کنید که بسیاری از مسائل خوب به راحتی حل نمی شوند بنابر این اگر در حل هر مساله ای موفق نشدید، ناامید نشوید. برای حل مسائل تلاش کنید هر چند اگر ساعتها و روزها وقت شما را بگیرد. از وقتهای اضافی (هنگام پیاده روی - ایستادن در صفهای مختلف اتوبوس، خرید نان و ...) برای حل مسائل و فکر کردن روی آنها استفاده کنید. روی مسائل کتابهای درسی خود خوب فکر کنید و برای حل آنها وقت بگذارید اما به آنها اکتفا نکنید. همیشه یک مساله جدید برای حل در ذهنتان داشته و به دنبال مسائل جدید باشید. از هیچ مساله ای نترسید. از مسائل مربوط به المپیادهای سالهای گذشته کشوری و بین المللی اطلاع داشته باشید و اگر فرصت کردید راه حل آنها را نیز پیدا کنید. در کل سعی کنید دایرة المعارف مسائل ریاضی ذهنتان را -یعنی مجموعه مسائلی که دیده اید نه مسائلی که حل کرده اید- دائماً توسعه دهید. اگر چند ماه خودتان را به این کارها عادت دهید، مسائل کتابهای درسی - و نتیجتاً تستهای کنکور- برایتان کاملا پیش پا افتاده خواهد شد. به امید خدا در همین تایپیک به بعضی از کتابهای معتبر مساله نیز اشاره خواهد شد.

4- مسائل جدید طراحی کنید. متن بعضی از مسائل کتاب را (بعد از حل آنها) به گونه ای مناسب تغییر دهید و سپس آنرا حل کنید. مثلا صورت و مخرج مساله را با هم عوض کنید، مثبها را منفی و منفی ها را مثبت کنید، اعداد را تغییر دهید، به مساله یک رادیکال اضافه یا کم کنید، اگر مساله ای با یک فرض به شما داده شده است فرض را بردارید و بررسی کنید که آیا مساله بدون آن فرض نیز درست یا نه، اگر درست است آنرا بدون آن فرض حل کنید و اگر درست نیست برای آن، مثال نقض ارائه کنید. بررسی کنید که آیا عکس مسائلی که به صورت شرطی داده شده اند درست است یا نه و ...

5- روی بعضی از مسائل گروهی کار کنید. می توانید چند مساله (از کتاب یا خارج آن) انتخاب و بین خود تقسیم و در فرصتی که معین می کنید روی آنها کار کنید و سپس راه حلها را با یکدیگر بررسی نمایید و اگر توانستید راه حل این مسائل را با معلمین خود نیز در میان بگذارید.

6- از مطالعه مجلات ریاضی (همانند «مجله برهان» و یا «رشد ریاضی») غافل نشوید. این مجلات تاثیر بسیار خوبی روی خواننده خود می گذارند.

7- اما آخرین پیشنهاد در این قسمت: در مسابقات علمی شرکتی فعال داشته باشید، چه در آنها برنده شوید، چه نشوید. اگر در شهر شما دانش آموزانی هستند که در مسابقات ریاضی موفق بوده اند، با آنها ارتباط علمی برقرار و از تجربیاتشان استفاده کنید. در حد توانتان در سمینارهای علمی مدرسه، شهر و ... شرکت کنید و اگر می توانید برای این سمینارها مقاله ای بنویسید و در آنها درباره کارتان سخنرانی کنید. گاهی هم به دانشگاههای شهرتان سری بزنید و اگر اجازه دادند از کتابخانه و فضای علمی آنجا استفاده کنید.

پاسخ سوال (ب):

به راستی آیا واقعاً راه میانبری در تست زنی- همانگونه که بسیاری از موسسات کنکور ادعا می کنند - وجود دارد؟ آیا واقعاً می توان دانش آموزی را که پایه علمی او بسیار ضعیف است با این به اصطلاح «روشهای من درآوردی» به رتبه های اول کنکور رساند و قبولی او را در دانشگاه تضمین کرد؟! مطمئن باشید که چنین راهی وجود ندارد! دلیل آن نیز -غیر از تجربه های این حقیر و سایر همکارانم- رتبه اولی های کنکور هستند. سالهاست که بسیاری از رتبه های اول کنکور در مصاحبه های خود بیان می کنند که حتی یک کلاس کنکور هم ندیده اند و عامل موفقیت خود را بعد از توکل بر خدا و زحمات پدر و مادر و معلمینشان، تلاش و کوشش خود می دانند و معمولا به این نکته هم اشاره می کنند که از اولین روزهای ورورد به دبیرستان درسها را خوب و عمیق خوانده اند و آنرا به روزهای نزدیک کنکور حواله نکرده اند. متاسفانه تبلیغات کاملاً حساب شده ای که سالهاست موسسات کذایی کنکور حتی در رادیو و تلوزیون به راه انداخته اند کار خود را کرده و باعث تغییر ذائقه علمی خانواده ها شده است به طوریکه با نهایت تاسف بسیاری از پدر و مادران عزیز ما قبولی فرزندانشان در کنکور را مساوی شرکت آنها در موسسات کنکور می دانند که البته این تغییر ذائقه به نفع جیب مبارک این موسسات هم تمام شده است و بد نیست بدانید که طبق آماری، مجموع پولی که موسسات کنکور کشور سالیانه به جیب می زنند تقریبا برابر است با پولی که از صنعت نفت عاید کشور می شود(!!!) بنابر این بهتر است نام بعضی از این موسسات را «کارخانجات صنایع کنکور» بگذاریم. این را به تجربه خدمتتان عرض می کنم - و با تحقیق کوچکی خودتان نیز به آن دست می یابید- که اکثریت کسانی که نامشان در بروشورهای تبلیغاتی یا در تبلیغات صدا و سیمای موسسات کنکور به عنوان قبولیهای رتبه های اول دانشگاه از آن موسسه آورده می شود از دانش آموزان باسواد و معدل بالای دبیرستان هستند که اگر در آن موسسه شرکت هم نمی کردند در دانشگاه قبول می شدند. فکر می کنید چند درصد از این دانش آموزان از آنهایی بوده اند که سطح معلومات علمیشان از متوسط به پایین است و با معجزه این آقایان به دانشگاه راه یافته اند؟! اگر هم چنین افرادی در میان قبولیها پیدا شود اولا درصدشان بسیار پایین است، ثانیاً خودشان هم بسیار تلاش کرده اند و اگر همین تلاش را بیرون از موسسه می کردند چه بسا رتبه بهتری می آوردند. حتی اگر چنین افرادی به طور کاملا تصادفی و به قول خودشان با کلکهای کنکوری- و یا علل دیگری که درست نیست در اینجا درباره آنها صحبت کنیم - در دانشگاه قبول شده اند تازه اول بدبختی آنهاست. اینها معمولا در دانشگاه دوام نمی آورند و یا با هزار بدبختی و فلاکت فارغ التحصیل می شوند. حال با این مقدمه طولانی سعی می کنیم به سوال قسمت (ب) پاسخ دهیم:

1- مطمئن شوید که دروس ریاضی را به طور عمقی مطالعه کرده اید، روی مسائل ریاضی داخل و خارج کتاب به اندازه لازم فکر کرده اید و موفق به حل بسیاری از آنها شده اید. از لحاظ روانی خود را متقاعد کنید که قوت و قدرت علمی لازم را برای رقابت با دیگران در مسابقه ای به نام کنکور به دست آورده اید. به طور خلاصه مطمئن شوید که در حد توانتان به مراحل قسمت (الف) -که در بالا به آنها اشاره شد - عمل کرده اید. توجه کنید که این مرحله بسیار مهم است و بدون عبور از این مرحله به هیچ عنوان نباید وارد مراحل بعدی شوید.

2- تستهای «خام» ریاضی ده سال اخیر کنکور سراسری را تهیه کنید. به عبارت «خام» توجه کنید. تستها دقیقا باید همانهایی باشند که در کنکور سراسری بدون هیچ گونه دخل و تصرفی به داوطلبان داده شده است. در بعضی از کتابها تستها به صورت طبقه بندی شده و موضوعی هستند. این گونه کتابها و جزوات برای این مرحله مناسب نیستند.

3- بعد از تهیه این تستها، سوالات کنکور دو سال اخیر را کنار بگذارید به گونه ای که جلوی چشمان شما نباشد. به اصطلاح آنها را در قرنطینه بگذارید. سپس چند روزی با فرصت مناسبی که برای خود کنار می گذارید، تستهای هشت سال باقیمانده را موضوع بندی کنید. به طور مثال سوالات سال 75 کنکور را بردارید و از تست اول شروع کنید. با دقت تمام تعیین کنید که این تست مربوط به کدام کتاب درسی و کدام موضوع و فصل آن کتاب است و این موارد را یادداشت کنید. (در این مرحله لازم نیست که خود تست را حل کنید.) همین کار را تا تست آخر انجام دهید. بعد از اتمام این کار، تستهای هم موضوع را کنار یکدیگر در دفتری یادداشت کنید و سپس برای خود آماری از این موضوعات تهیه کنید که مثلا چند درصد از تستها در موضوع توابع، حد و پیوستگی، مشتق ، انتگرال ، خط و صفحه، ماتریسها، مثلثات، محاسبات لگاریتمی و ... هستند. همین کارها را برای سالهای دیگر نیز تکرار کنید و در آخر، درصد موضوعی تستهای این هشت سال را محاسبه کنید. حال با نگاهی کلی می توانید حدس بزنید که از کدام موضوع بیشتر سوال طرح شده است و باید روی کدام موضوعات بیشتر کار کنید و اگر ضعفی دارید برطرف نمایید.

4- حالا شروع کنید و تستهای هم موضوعی که کمترین درصد آمار شما را دارند حل کنید. در حل تستها عجله نکنید. آنرا به عنوان یک مساله نگاه کنید نه به عنوان تست. مطمئن باشید که اگر درسها را به خوبی خوانده باشید و روی مسائل مختلف فکر کرده باشید، حل این تستها برایتان به هیچ عنوان سخت نخواهد بود. اگر موفق به حل تست شدید ، حل آنرا هم یاداشت کنید. اگر نتوانستید تست را حل کنید بلافاصله به جواب آن مراجعه نکنید و برای حل این تست تلاش کنید حتی اگر یکساعت هم وقت شما را بگیرد. اگر باز هم موفق نشدید به راه حل آن مراجعه کنید و اگر راه حلی در اختیارتان نبود وارد حل تست بعدی شوید و بعدا روش حل تستی که از عهده حل آن بر نیامده اید از معلمین یا دوستانتان بپرسید و روش آنرا هم در دفتر یاداشت کنید. به هیچ عنوان از اینکه نتوانسته اید تست را در چند ثانیه حل کنید مایوس نشوید. سرعت تست زنی شما با سماجت شما در حل تستهای اولیه افزایش خواهد یافت. همین کار را برای موضوعات دیگر نیز که درصد بالاتری دارند به ترتیب انجام دهید.
این روش شما را مجبور خواهد کرد که دائماً به کتاب و دفترتان مراجعه کنید و همین کار تجربه تست زنی شما را افزایش خواهد داد و در جلسه کنکور به دردتان خواهد خورد. شاید این مرحله روزها و هفته ها و شاید ماهها طول بکشد، اما بسیار کارساز است و ترس شما را از مواجهه با تستهای مشکل تقریبا از بین می برد.

5- بعد از اینکه مرحله چهارم به اتمام رسید، این مرحله را یکبار دیگر تکرار کنید. این بار سرعت حل تستها باید بیشتر شده باشد زیرا قبلا آنها را حل کرده اید. مطمئن شوید که جواب همه تستها را می دانید و راه حلها را هم کاملا مرور کرده اید.

6- حال شریط جلسه کنکور را برای خودتان در خانه یا کتابخانه های عمومی و یا جاهای دیگر مهیا کنید. مکان ساکتی که حواس شما را پرت نکند. یکی از تستهای کنار گذاشته شده را از قرنطینه خارج کنید و با توجه به زمانی که برای شما در کنکور تعیین می شود، تستها را حل کنید. در آخر ببینید چند درصد تستها را درست حل کرده اید و علت اینکه تستی را درست حل نکرده اید چیست. سپس با رعایت موضوع، روش حل تستها را در دفتر مربوطه بنویسید.

7- تمام مرحله 6 را یکبار دیگر با تست کنار گذاشته شده دوم انجام دهید و بار دیگر خودتان را بسنجید. در این مرحله باید سرعت تست زنی شما و تعداد تستهای درست، افزایش یافته باشد.

8- در این مرحله - البته در صورت داشتن وقت کافی- تستهای جدید طرح کنید و بعد از اینکه به تعداد مناسبی رسید، با این تستها از خودتان امتحان بگیرید و سرعت و مهارت خود را بسنجید.

9- بعد از طی مراحل بالا مجازید که کتابهای معتبر تست را تهیه کنید و با تستهای بیشتری آشنا شوید. آموزش و پرورش کتابهای تست خوبی در موضوعات مختلف منتشر کرده است که می توانید از آنها استفاده کنید. البته کتابهای خوب تست منحصر به این کتابها نیست.

10- در چند روز مانده به کنکور، مطالعه را متوقف و فقط خلاصه دروس و مطالبی که به طور موضوعی در دفتر حل تستها یادداشت کرده اید، مرور کنید و جداً از خسته کردن خود بپرهیزید که خستگی در جلسه امتحان بسیاری از تلاشها یتان را بر باد خواهد داد. برادرانه و خاضعانه به خواهران و برادران مومن خودم توصیه می کنم که با وضو و نیز با صلوات بر محمد و آل محمد و با توکل برخدا و توسل به اهلبیت عصمت و طهارت (صلوات الله علیهم اجمعین) در جلسه کنکور حاضر شوید و مطمئن باشید که نتیجه زحمات خود را خواهید دید و شهد شیرین موفقیت را خواهید چشید، انشاءالله.

موفق و موید و پیروز باشید.

جمعه 24 شهریور 1385

=================================================

سوال دوم: براي المپياد رياضي بايد چه منابعي رو مطالعه كرد؟


به چند منبع معتبر اشاره می کنم (توجه داشته باشید که فرض بر این است که پایه علمی شما خوب است و ضعف خاصی در کتابهای درسی دبیرستان و پیش دانشگاهی ندارید. در غیر این صورت منابع زیر خیلی به دردتان نخواهد خورد):

1- کتابهای کار و راهنمای مطالعه دانش آموز (وزارت آموزش و پرورش - انتشارات فاطمی)

2- سری کتابهای کوچک ریاضی (انتشارات مدرسه)

3- المپیاد ریاضی در ایران- تالیف دکتر عبادلله محمودیان (موسسه علمی انتشارات علمی دانشگاه صنعتی شریف)

4- حل مساله از طریق مساله تالیف لورن سی. لارسن و ترجمه آقای علی ساوجی (انتشارات فاطمی)

5- پانصد مساله ریاضی پیکارجو تالیف باربو-کلامکین-موزر و ترجمه مهران اخباریفر (انتشارات فاطمی)

6- از اردوش تا کی یف تالیف هانس برگر و ترجمه علی ساوجی (انتشارات فاطمی)

7- کتابهای تکمیلی ریاضیات ( این کتابها در مراکز استعداد های درخشان به دانش آموزان تحویل داده می شود و یکی از منابع تدریس دبیران در این مراکز است) (انتشارات سمپاد)

8- کتاب روشهای جبر - تالیف استاد پرویز شهریاری (انتشارات امیرکبیر) (لازم به ذکر است که تمامی کتابهای تالیف شده یا ترجمه شده توسط این استاد گرانقدر و چهره ماندگار ریاضی منابعی عالی برای مطالعه دانش آموزان است.)

9- سری کتابهای ریاضیات پیش دانشگاهی (انتشارات مرکز نشر دانشگاهی)

10- کتاب «اثبات بدون کلام» تالیف راجر ب. نلسن ترجمه خانم سپیده چمن آرا (انتشارات فاطمی)

موفق باشید.

جمعه 24 شهریور 1385

=================================================

سوال سوم: میشه کتابایی رو برای شروع ریاضی از آغاز تا ... معرفی کنین. میخوام ریاضی ام که از اول مشکل داشتم رو حل کنم . چون تصمیم گرفتم ریاضیم تــــــــــــــــــــــــ ــــــــــــوپ بشه.

بهترین کتابها همان کتابهای درسی هستند. منتهی باید به روشی درست مطالعه شوند. در پاسخ به سوالات اول و دوم در اینباره و نیز کتابهای دیگر صحبت شده است.

موفق باشید.

جمعه 24 شهریور 1385

=================================================
سوال چهارم: سلام
رياضي بسيار سخته :puke: :angry:
شبها هم براش كابوس مي بينم :sad:
يكي بياد يك چيز خوب در موردش بگه كه من با رياضي بتونم كنار بيام ;)
چون عجيب گيرشم :sad:
با تشكر

دوست عزیزم، شنا بلدی یا نه؟ می دونی چه جوری باید شنا یاد گرفت؟ می شه خیلی ساده و خودمونی برام بگی که اگر یه نفر بخواد شنا یاد بگیره اما از غرق شدن، حسابی می ترسه باید چی کار کنه؟ خودش را در کنار نجات غریق به آب می زنه و ترسش رو از آب کمتر و کمتر می کنه. درسته که دفعه اول بسیار سخته، اما این کار رو حتما باید انجام بده. باور کن ریاضی ترسناکتر از اولین شنا در یک استخر شش متری نیست. تعداد کسایی که با ریاضیات کنار اومدن و از اون لذت می برن از کسایی که از ریاضیات خوششون نمی آد، اصلا کمتر نیست. این خودش نشون می ده که ریاضی اصالتا ترسناک نیست و باید دل رو به دریا بزنی و برای یکبار هم که شده به طور استاندارد ریاضی خوندن رو شروع کنی.

موفق باشید.

abay
07-09-2006, 15:35
[ برای مشاهده لینک ، لطفا با نام کاربری خود وارد شوید یا ثبت نام کنید ]
سلام دوست عزیز
خیلی ممنون از تذکری که دادین حالا به طور اتفاقی با این روشی که من مسئله را حل کرده ام جواب بدست آمده ولی روش حل قسمت آخر درست نیست وما نمی توانیم[ برای مشاهده لینک ، لطفا با نام کاربری خود وارد شوید یا ثبت نام کنید ] رااز[ برای مشاهده لینک ، لطفا با نام کاربری خود وارد شوید یا ثبت نام کنید ]کم کنیم زیرا چنین رابطه ای را نداریم[ برای مشاهده لینک ، لطفا با نام کاربری خود وارد شوید یا ثبت نام کنید ]

ali1234
07-09-2006, 16:02
درسته چنين رابطه اي وجود نداره

rap.vartan
07-09-2006, 16:42
چند تا سوال دارم
كمكم كنيد
1. 5 مهره ي سفيد يكسان و10 مهره سياه يكسان داريم . چند حالت وجود دارد كه بعد از مهره ي سفيد حداكثر يك مهره ي سياه باشد؟

2.يك اتوبوس با 40مسافر از نقطه ي A حركت مي كند در 6 ايستگاه بين راهي 35 نفر را پياده مي كند و 5 نفر ديگر را در ايستگاه آخر . چند حالت وجود دارد ؟

3. سه عدد A B C مفروضند به طوري كه حاصل جمع آن ها 1 است و حاصل جمع مربع آن ها 1 است و حاصل مكعبات آن ها 1 است . حاصلضرب سه عدد چند است ؟

4. يك قورباغه مي خواهند از روي سنگ 1 به سنگ 7 برود به شرطي كه وقتي كه روي 1 قرار دارد فقط يك سنگ بتواند بپرد (مثلا : اگر روي 4 باشه مي تونه 1 يا 2 يا 3 سنگ را بپرد )در چند حالت مي تواند از سنگ 1 به سنگ 7 برسد ؟

مرسي
ممنون
يك سوال از شما آقاي مفيدي
براي المپياد رياضي بايد چه مطالبي را مطالعه كرد

mofidy1
08-09-2006, 16:38
با سلام

فرض کنید 1=x+y+z که y، x و z اعداد حقیقی نامنفی هستند. ثابت کنید:


[ برای مشاهده لینک ، لطفا با نام کاربری خود وارد شوید یا ثبت نام کنید ]

موفق باشید.

ارسال متن: شنبه 11 شهریور 1385

با سلام

از دوست خوبم abay که در پست 295 به حل مساله پرداختند ممنونم. براي ديدن حل کامل مساله به لينك زير مراجعه كنيد:

[ برای مشاهده لینک ، لطفا با نام کاربری خود وارد شوید یا ثبت نام کنید ]

(این مساله در المپیاد بین المللی ریاضی سال 1983 مطرح شده بود.)

موفق باشید.

ارسال متن: جمعه 17 شهریور 1385

mofidy1
09-09-2006, 00:57
با سلام

مقدار عبارت زیر را - بدون استفاده از ماشین حساب - به دست آورید:


[ برای مشاهده لینک ، لطفا با نام کاربری خود وارد شوید یا ثبت نام کنید ]

موفق باشید.

ارسال متن: شنبه 18 شهریور 1385

abay
09-09-2006, 21:25
با سلام

از دوست خوبم abay که در پست 295 به حل مساله پرداختند ممنونم. براي ديدن حل کامل مساله به لينك زير مراجعه كنيد:

[ برای مشاهده لینک ، لطفا با نام کاربری خود وارد شوید یا ثبت نام کنید ]

(این مساله در المپیاد بین المللی ریاضی سال 1983 مطرح شده بود.)

موفق باشید.

ارسال متن: جمعه 17 شهریور 1385
سلام
آقای مفیدی یه کمی در موردحل مسئله توضیح دهید.
در ضمن آقای مفیدی برای نوشتن فرمولهای ریاضی شما یک نرم افزاری به نامwinEdtبه بنده معرفی کرده بودیداگه کرک یا شماره سریال این برنامه را دارید لطفا"لینکی برای دانلودشون بزارید آخه من هر چی گشتم سریال یا کرک این ورژنو پیدا نکردم اگه میشه یه توضیحی هم در مورد نحوه ی کار با این برنامه واین که در کجا فرمولهای ریاضی را با زبان texبنویسیم رو بدیدو یک سوال دیگه اینکه من کم وبیش با زبانtex آشنا شده ام اما بعضی از دستورها رو نمی تونم بنویسم یکی اینکه آیا با texمیشه انتگرال معین را نوشت یعنی عدد ها یی که دربالا وپایین علامت انتگرال هست رو نوشت ویا چنین چیزی در مورد علامت سیگما آیا میشه محدوده ها را مشخص کرد وسوال دیگه اینکه چه جوری میشه تابع های چند ضابطه ای رو با این زبان نوشت مثلا" جلوی آکلاد چه جوری سه سطر فرمول بنویسیم.

mofidy1
10-09-2006, 12:17
سلام
آقای مفیدی یه کمی در موردحل مسئله توضیح دهید.
در ضمن آقای مفیدی برای نوشتن فرمولهای ریاضی شما یک نرم افزاری به نامwinEdtبه بنده معرفی کرده بودیداگه کرک یا شماره سریال این برنامه را دارید لطفا"لینکی برای دانلودشون بزارید آخه من هر چی گشتم سریال یا کرک این ورژنو پیدا نکردم اگه میشه یه توضیحی هم در مورد نحوه ی کار با این برنامه واین که در کجا فرمولهای ریاضی را با زبان texبنویسیم رو بدیدو یک سوال دیگه اینکه من کم وبیش با زبانtex آشنا شده ام اما بعضی از دستورها رو نمی تونم بنویسم یکی اینکه آیا با texمیشه انتگرال معین را نوشت یعنی عدد ها یی که دربالا وپایین علامت انتگرال هست رو نوشت ویا چنین چیزی در مورد علامت سیگما آیا میشه محدوده ها را مشخص کرد وسوال دیگه اینکه چه جوری میشه تابع های چند ضابطه ای رو با این زبان نوشت مثلا" جلوی آکلاد چه جوری سه سطر فرمول بنویسیم.

با سلام

دوست عزیز در کدام قسمت حل مساله مشکل دارید. بفرمایید تا توضیح دهم.

قبلا عرض کرده بودم که نرم افزار WinEdt نرم افزاری کاملا حرفه ایست و مخصوص نوشتن مقالات تخصصی به زبان انگلیسی. استفاده از آن را خیلی توصیه نمی کنم. البته اگر باز بر استفاده از آن اصرار دارید بنده در خدمتم و آموزش خلاصه ای از آن را در اتاق خواهم گذاشت. بنده استفاده از MathType را به شما توصیه می کنم که البته قبلا بارها درباره آن صحبت کرده ام.

ذیلا چند دستور tex را که خواسته بودید می آورم. آنها را copy-paste کنید و نتیجه را ببینید:

1- انتگرال سینوس از 0 تا pi/2

\int\limits_0^{\frac{\pi }
{2}} {\sin (x)dx}

2- مجموع n(n+2)/2 از n=0 تا n=10

\sum\limits_{n = 0}^{10} {\frac{{n(n + 1)}}
{2}}

3- یک مثال از تابع چند ضابطه ای - تعریف قدر مطلق x

|x| = \left\{ \begin{gathered}
x{\text{ }}x > 0 \hfill \\
0{\text{ }}x = 0 \hfill \\
- x{\text{ }}x < 0 \hfill \\
\end{gathered} \right.

موفق باشید.

eh_mn
12-09-2006, 11:23
سلام

آقاي مفيدي لطفا مرا در مورد اين مسئله راهنمايي كنيد:
فرض كنيم G يك گروه باشد. زيرگروه H از G را مشخصه گوييم هرگاه براي هر خودريختي مانند [ برای مشاهده لینک ، لطفا با نام کاربری خود وارد شوید یا ثبت نام کنید ] از G داشته باشيم[ برای مشاهده لینک ، لطفا با نام کاربری خود وارد شوید یا ثبت نام کنید ]
تقريبا واضح است كه هر زيرگروه مشخصه ، نرمال نيز هست. حال گروهي را معرفي كنيد كه زيرگروهي نرمال داشته باشد كه مشخصه نباشد.

موفق باشید

mofidy1
12-09-2006, 13:49
سلام

آقاي مفيدي لطفا مرا در مورد اين مسئله راهنمايي كنيد:
فرض كنيم G يك گروه باشد. زيرگروه H از G را مشخصه گوييم هرگاه براي هر خودريختي مانند [ برای مشاهده لینک ، لطفا با نام کاربری خود وارد شوید یا ثبت نام کنید ] از G داشته باشيم[ برای مشاهده لینک ، لطفا با نام کاربری خود وارد شوید یا ثبت نام کنید ]
تقريبا واضح است كه هر زيرگروه مشخصه ، نرمال نيز هست. حال گروهي را معرفي كنيد كه زيرگروهي نرمال داشته باشد كه مشخصه نباشد.

موفق باشید

با سلام

آقای منبتی قضیه بسیار مهمی در نظریه گروهها وجود دارد به این مضمون که اگر گروه متناهی G شامل هیچ زیرگروه مشخصه ی غیر بدیهی نباشد آنگاه یا 1=G یا G گروهی ساده است یا G حاصلضرب مستقیمی از گروههای ساده یکریخت است و بالعکس. بنابر این به طور مثال 4-گروه کلاین را (که حاصلضرب مستتقیم دو Z_2 است) در نظر بگیرید. این گروه آبلی است و لذا هر زیرگروه آن نرمال، اما تمامی زیرگروههای مرتبه 2 آن غیر مشخصه هستند. (البته این را می توانید مستقیما هم بررسی کنید.)

موفق باشید.

cool_lizard
12-09-2006, 18:45
سلام من اين مساله هفته شانزدهم رو حل كردم ولي برنامه نوشتنشو ندارم اگه ميشه آدرس جايي كه ميشه اين برنامه رو دانلود كرد بنويسيد تا منم بتونم پاسخهامو بنويسم با تشكر

bimo
13-09-2006, 11:53
يه سوال داشتم اونم اينه كه آيا تابعي وجودداره كه ما مقدار توان و به اون بديم و عدد بخوايم؟
يعني منظورم اينه كه توابع زير رو همه رو باهم داشته باشه:

f(x)=x , f(x)=x^2 , f(x)=x^3 ,...
يعني تابعي براي تك تكشون وجود داره كه فرمولشون تقريباَ مثل هم باشه و فقط يه كم با هم فرق كنن؟ ( البته تابع مي خوام فقط به صورت ضرب و جمع باشه نه تواندار)

mofidy1
13-09-2006, 12:00
سلام من اين مساله هفته شانزدهم رو حل كردم ولي برنامه نوشتنشو ندارم اگه ميشه آدرس جايي كه ميشه اين برنامه رو دانلود كرد بنويسيد تا منم بتونم پاسخهامو بنويسم با تشكر


با سلام

در دو پست زیر توضیحات لازم آورده شده است:

[ برای مشاهده لینک ، لطفا با نام کاربری خود وارد شوید یا ثبت نام کنید ]

[ برای مشاهده لینک ، لطفا با نام کاربری خود وارد شوید یا ثبت نام کنید ]

اگر باز مشکلی داشتید اطلاع دهید. منتظر راه حل مساله هفته شانزدهم هستیم.

موفق باشید.

rap.vartan
13-09-2006, 12:11
جناب مفيدي عزيز
براي المپياد رياضي بايد چه منابعي رو مطالعه كرد؟

مرسي
مرسي
و
ممنون

mofidy1
13-09-2006, 13:27
جناب مفيدي عزيز
براي المپياد رياضي بايد چه منابعي رو مطالعه كرد؟

مرسي
مرسي
و
ممنون

با سلام

به چند منبع معتبر اشاره می کنم (توجه داشته باشید که فرض بر این است که پایه علمی شما خوب است و ضعف خاصی در کتابهای درسی دبیرستان و پیش دانشگاهی ندارید. در غیر این صورت منابع زیر خیلی به دردتان نخواهد خورد):

1- کتابهای کار و راهنمای مطالعه دانش آموز (وزارت آموزش و پرورش - انتشارات فاطمی)

2- سری کتابهای کوچک ریاضی (انتشارات مدرسه)

3- المپیاد ریاضی در ایران- تالیف دکتر عبادلله محمودیان (موسسه علمی انتشارات علمی دانشگاه صنعتی شریف)

4- حل مساله از طریق مساله تالیف لورن سی. لارسن و ترجمه آقای علی ساوجی (انتشارات فاطمی)

5- پانصد مساله ریاضی پیکارجو تالیف باربو-کلامکین-موزر و ترجمه مهران اخباریفر (انتشارات فاطمی)

6- از اردوش تا کی یف تالیف هانس برگر و ترجمه علی ساوجی (انتشارات فاطمی)

7- کتابهای تکمیلی ریاضیات ( این کتابها در مراکز استعداد های درخشان به دانش آموزان تحویل داده می شود و یکی از منابع تدریس دبیران در این مراکز است) (انتشارات سمپاد)

8- کتاب روشهای جبر - تالیف استاد پرویز شهریاری (انتشارات امیرکبیر) (لازم به ذکر است که تمامی کتابهای تالیف شده یا ترجمه شده توسط این استاد گرانقدر و چهره ماندگار ریاضی منابعی عالی برای مطالعه دانش آموزان است.)

9- سری کتابهای ریاضیات پیش دانشگاهی (انتشارات مرکز نشر دانشگاهی)

10- کتاب «اثبات بدون کلام» تالیف راجر ب. نلسن ترجمه خانم سپیده چمن آرا (انتشارات فاطمی)

موفق باشید.

abay
15-09-2006, 00:19
با سلام

دوست عزیز در کدام قسمت حل مساله مشکل دارید. بفرمایید تا توضیح دهم.

قبلا عرض کرده بودم که نرم افزار WinEdt نرم افزاری کاملا حرفه ایست و مخصوص نوشتن مقالات تخصصی به زبان انگلیسی. استفاده از آن را خیلی توصیه نمی کنم. البته اگر باز بر استفاده از آن اصرار دارید بنده در خدمتم و آموزش خلاصه ای از آن را در اتاق خواهم گذاشت. بنده استفاده از MathType را به شما توصیه می کنم که البته قبلا بارها درباره آن صحبت کرده ام.
موفق باشید.
باسلام وخسته نباشید خدمت آقای مفیدی
در مورد سوال اشکال من یکی همین فرضی که شما در حل مسئله بکار برده ایدیه توضیحی در این مورد میخواستم بفرماییدواشکال دیگه قسمت[ برای مشاهده لینک ، لطفا با نام کاربری خود وارد شوید یا ثبت نام کنید ]است این عبارت چرا کوچکتر یا مساوی صفر است.درمورد نرم افزار WinEdt اگه آموزشی بدیدخیلی ممنون میشم

HITLER_fashist
15-09-2006, 05:46
میشه کتابایی رو برای شروع ریاضی از آغاز تا ... معرفی کنین. میخوام ریاضی ام که از اول مشکل داشتم رو حل کنم . چون تصمیم گرفتم ریاضیم تــــــــــــــــــــــــ ــــــــــــوپ بشه. و اگه لطف کنین به میلم یا مسنجرم بفرستیم واقعا ممنونتون میشم.
soroshking@gmail.com
soroshking@yahoo.com

mofidy1
15-09-2006, 13:19
باسلام وخسته نباشید خدمت آقای مفیدی
در مورد سوال اشکال من یکی همین فرضی که شما در حل مسئله بکار برده ایدیه توضیحی در این مورد میخواستم بفرماییدواشکال دیگه قسمت[ برای مشاهده لینک ، لطفا با نام کاربری خود وارد شوید یا ثبت نام کنید ]است این عبارت چرا کوچکتر یا مساوی صفر است.درمورد نرم افزار WinEdt اگه آموزشی بدیدخیلی ممنون میشم

با سلام

دوست عزیز چون مجموع این سه عدد، یک است نمی توانند همگی از 1/2 بزرگتر باشد و بالاجبار باید یکی از آنها از 1/2 کمتر یا مساوی باشد. حال چون عبارت اصلی مساله نسبت به y، x و z متقارن است (اگر y، x و z را به یکدیگر تبدیل کنیم مساله عوض نمی شود)، می توان به دلخواه فرض کرد که z از 1/2 کمتر یا مساوی است.

عبارتی که پرسیدید نامنفی است. توجه کنید که در حل مساله یک منفی پشت این عبارت وجود دارد.

درمورد نرم افزار WinEdt نیز به روی چشم. در اولین فرصت آموزش کوتاهی از آن را خواهم گذاشت. راستی دستورات tex ی که برایتان نوشته بودم به دردتان خورد؟

موفق باشید.

mofidy1
15-09-2006, 13:25
میشه کتابایی رو برای شروع ریاضی از آغاز تا ... معرفی کنین. میخوام ریاضی ام که از اول مشکل داشتم رو حل کنم . چون تصمیم گرفتم ریاضیم تــــــــــــــــــــــــ ــــــــــــوپ بشه. و اگه لطف کنین به میلم یا مسنجرم بفرستیم واقعا ممنونتون میشم.
soroshking@gmail.com
soroshking@yahoo.com

با سلام

بهترین کتابها همان کتابهای درسی هستند. منتهی باید به روشی درست مطالعه شوند. در لینک زیر در اینباره و نیز کتابهای دیگر صحبت شده است:

[ برای مشاهده لینک ، لطفا با نام کاربری خود وارد شوید یا ثبت نام کنید ]

موفق باشید.

mofidy1
15-09-2006, 21:40
با سلام

مقدار عبارت زیر را - بدون استفاده از ماشین حساب - به دست آورید:


[ برای مشاهده لینک ، لطفا با نام کاربری خود وارد شوید یا ثبت نام کنید ]

موفق باشید.

ارسال متن: شنبه 18 شهریور 1385

با سلام

برای دیدن حل مساله هفته شانزدهم به لینک زیر مراجعه فرمایید:

[ برای مشاهده لینک ، لطفا با نام کاربری خود وارد شوید یا ثبت نام کنید ]

این مساله در المپیاد ریاضی کشور در سال 1367 مطرح شده بود. (کتاب المپیاد ریاضی در ایران - تالیف دکتر عبادالله محمودیان)

ارسال متن: جمعه 25 شهریور 1385

موفق باشید.

abay
16-09-2006, 01:22
سلام
آقای مفیدی اگه میشه یه نرم افزاری برای رسم شکلهای هندسی ونمودارها معرفی کنید البته شما یک نرم افزار به نامwinGCLCرا دریکی از پست ها معرفی کرده بودیداما من هر چه جستجو کردم نتونستم پیدا کنم اگه براتون مقدور بود لینک این برنامه را برای دانلود بذارید یا حداقل جاییکه میشه این برنامه را دانلود کرد رو بگید.

abay
16-09-2006, 01:24
با سلام

برای دیدن حل مساله هفته شانزدهم به لینک زیر مراجعه فرمایید:

[ برای مشاهده لینک ، لطفا با نام کاربری خود وارد شوید یا ثبت نام کنید ]

این مساله در المپیاد ریاضی کشور در سال 1367 مطرح شده بود. (کتاب المپیاد ریاضی در ایران - تالیف دکتر عبادالله محمودیان)

ارسال متن: جمعه 25 شهریور 1385

موفق باشید.
سلام
آقای مفیدی این سواله هفته شانزدهم که درست هم سن من است رو من دیروز دیدم چه زود جوابش دادید
حالا بگذریم من چند تا سوال ساده داشتم امیدوارم شما ودوستان بتونید رفع اشکال کنیدیکی از سوالاتم مربوط به اصل لانه کبوتره که اینه:کتابی دارای 900صفحه است هر صفحه اش بین 150تا250کلمه داردحداقل چند صفحه دراین کتاب است که عدد کلمات آنها برابر است ؟
2)اثبات کنید:الف)[ برای مشاهده لینک ، لطفا با نام کاربری خود وارد شوید یا ثبت نام کنید ]
ب)اگر در معادله ی درجه ی دوم با ضرایب گویا یکی از ریشه ها برابر [ برای مشاهده لینک ، لطفا با نام کاربری خود وارد شوید یا ثبت نام کنید ]ریشه ی دوم آن[ برای مشاهده لینک ، لطفا با نام کاربری خود وارد شوید یا ثبت نام کنید ]خواهد بود.
3)سوال سوم در مورد حل یک انتگرال است [ برای مشاهده لینک ، لطفا با نام کاربری خود وارد شوید یا ثبت نام کنید ]میدانیم روش عادی حل این مسئله چنین است[ برای مشاهده لینک ، لطفا با نام کاربری خود وارد شوید یا ثبت نام کنید ]اما بنده با روش دیگری این مسئله را حل کردم[ برای مشاهده لینک ، لطفا با نام کاربری خود وارد شوید یا ثبت نام کنید ]اما دبیر ریاضی ما این روش را روش درستی برای حل چنین مسایلی قبول نمی کرد ودلیلش این بود که چنین روشی در حل مسایل معدودی مثل همینی که من حل کردم جواب می دهد وهمواره درست نیست ومثال دیگری آورد [ برای مشاهده لینک ، لطفا با نام کاربری خود وارد شوید یا ثبت نام کنید ]که نمی توان این مسئله با این روش حل کرد ودلیلش این بود که[ برای مشاهده لینک ، لطفا با نام کاربری خود وارد شوید یا ثبت نام کنید ]با[ برای مشاهده لینک ، لطفا با نام کاربری خود وارد شوید یا ثبت نام کنید ]برابر نیست البته این دلیل برای من قانع کننده بود اما اگراین دو عبارت را با هم برابر بگیریم وبا همین روش حل کنیم جواب انتگرال درست یعنی اگه مشتق بگیریم همین عبارت بدست می آید ومن میخواستم بدانم چرا چنین چیزی روی میدهد وآیا این روشی که من دارم روش غلطی است و نمیتوان برای حل چنین مسایلی به آن استناد کرد یا نه؟ضمنا"من دوتا انتگرال دیگر اینجا می نویسم البته خودم آنها را حل کرده ام اما میخواستم راه حلهای دیگری که دوستان برای حل این مسایل دارند را ببینم اگه دوست داشتید حل کنید.[ برای مشاهده لینک ، لطفا با نام کاربری خود وارد شوید یا ثبت نام کنید ]و[ برای مشاهده لینک ، لطفا با نام کاربری خود وارد شوید یا ثبت نام کنید ]

Dr.Adeli
16-09-2006, 03:24
کمک کمک

من فرمول واریانس و انحراف معیار رو می خوام( بعد 20-15 سال حق بدین یادم نیاد!!)
خیلی حیاتیه
کدوم رادیکال داشت؟؟؟ :puke:

mofidy1
16-09-2006, 14:22
کمک کمک

من فرمول واریانس و انحراف معیار رو می خوام( بعد 20-15 سال حق بدین یادم نیاد!!)
خیلی حیاتیه
کدوم رادیکال داشت؟؟؟ :puke:

با سلام

فرمول اول میانگین، دومی واریانس و سومی انحراف معیار n تا متغیر تصادفی است:


[ برای مشاهده لینک ، لطفا با نام کاربری خود وارد شوید یا ثبت نام کنید ]

موفق باشید.

mofidy1
16-09-2006, 16:12
سلام
آقای مفیدی اگه میشه یه نرم افزاری برای رسم شکلهای هندسی ونمودارها معرفی کنید البته شما یک نرم افزار به نامwinGCLCرا دریکی از پست ها معرفی کرده بودیداما من هر چه جستجو کردم نتونستم پیدا کنم اگه براتون مقدور بود لینک این برنامه را برای دانلود بذارید یا حداقل جاییکه میشه این برنامه را دانلود کرد رو بگید.

با سلام

برای دانلود WinGCLC به صفحه زیر مراجعه کنید:

[ برای مشاهده لینک ، لطفا با نام کاربری خود وارد شوید یا ثبت نام کنید ]

فایل این برنامه به صورت zip است. آنرا از حالت فشرده خارج کنید و در درون پوشه wingclc روی فایل اجرایی آن کلیک کنید. بعد از اجرای این برنامه، به وسیله آن از درون یکی از پوشه ها به نام samples، یک فایل را به دلخواه open کنید و سپس در منوی اصلی روی علامتی که به صورت یک فلش عمودی است کلیک کنید تا برنامه اجرا شود. با مقایسه دستوراتی که در سمت چپ نوشته شده با شکلی که ظاهر می شود می توانید شکلهای جدیدی تولید کنید.

ضمنا دوست عزیز بنده به سوالات ریاضی هفته هر جمعه جواب می دهم و اینبار هم زود جواب ندادم. احتمالا شما سوال هفته شانزدهم را دیر متوجه شدید.

موفق باشید.

ali_hp
16-09-2006, 16:26
با سلام

فرمول اول میانگین، دومی واریانس و سومی انحراف معیار n تا متغیر تصادفی است:


[ برای مشاهده لینک ، لطفا با نام کاربری خود وارد شوید یا ثبت نام کنید ]

موفق باشید.
سلام
ایا در مورد n-1 موجود درمخرج فرمول دوم وسوم اطمینان دارید؟

mofidy1
16-09-2006, 18:23
سلام
ایا در مورد n-1 موجود درمخرج فرمول دوم وسوم اطمینان دارید؟

با سلام

بله مطمئن هستم. برای نمونه های گروه بندی شده مخرج این دو n است که البته فرمول آنها کمی فرق دارد و از فراوانی و نقطه وسط داده ها استفاده می شود.

راستی علی آقا چه خبر از دانشگاه؟ کجا و در چه رشته ای قبول شدید؟

mofidy1
16-09-2006, 22:32
با سلام

فرض کنید


[ برای مشاهده لینک ، لطفا با نام کاربری خود وارد شوید یا ثبت نام کنید ]

اگر برای هر x حقیقی داشته باشیم:


[ برای مشاهده لینک ، لطفا با نام کاربری خود وارد شوید یا ثبت نام کنید ]

ثابت کنید:


[ برای مشاهده لینک ، لطفا با نام کاربری خود وارد شوید یا ثبت نام کنید ]

موفق باشید.

ارسال متن: شنبه 25 شهریور 1385

Marichka
16-09-2006, 23:58
سلام دوست عزيز :)
سوالات خودتون رو در مورد رياضي در اين تاپيك:
[ برای مشاهده لینک ، لطفا با نام کاربری خود وارد شوید یا ثبت نام کنید ]

مطرح كنين لطفا آقاي مفيدي كمكتون مي كنن. :rolleye:

موفق باشيد

ali_hp
17-09-2006, 00:02
راستی علی آقا چه خبر از دانشگاه؟ کجا و در چه رشته ای قبول شدید؟
سلام
علوم رياضي شريف

mofidy1
17-09-2006, 00:26
با سلام

آقای حسین پوران قبولی شما را در دانشگاه صنعتی شریف آنهم در رشته ریاضی تبریک عرض می کنم. انشاءالله همیشه موفق و موید باشید.

ali_hp
17-09-2006, 00:33
با سلام

آقای حسین پوران قبولی شما را در دانشگاه صنعتی شریف آنهم در رشته ریاضی تبریک عرض می کنم. انشاءالله همیشه موفق و موید باشید.
متشکرم.

Dr.Adeli
17-09-2006, 00:46
ممنون دوست عزيز

eh_mn
17-09-2006, 02:04
با سلام

1. آقاي مفيدي از راهنمايي شما خيلي ممنونم.
2. آقاي حسين پوران من هم به نوبه خود قبولي شما را تبريك مي گويم. اميدوارم موفق باشيد.

abay
17-09-2006, 14:15
با سلام

برای دانلود WinGCLC به صفحه زیر مراجعه کنید:

[ برای مشاهده لینک ، لطفا با نام کاربری خود وارد شوید یا ثبت نام کنید ]

فایل این برنامه به صورت zip است. آنرا از حالت فشرده خارج کنید و در درون پوشه wingclc روی فایل اجرایی آن کلیک کنید. بعد از اجرای این برنامه، به وسیله آن از درون یکی از پوشه ها به نام samples، یک فایل را به دلخواه open کنید و سپس در منوی اصلی روی علامتی که به صورت یک فلش عمودی است کلیک کنید تا برنامه اجرا شود. با مقایسه دستوراتی که در سمت چپ نوشته شده با شکلی که ظاهر می شود می توانید شکلهای جدیدی تولید کنید.

ضمنا دوست عزیز بنده به سوالات ریاضی هفته هر جمعه جواب می دهم و اینبار هم زود جواب ندادم. احتمالا شما سوال هفته شانزدهم را دیر متوجه شدید.

موفق باشید.
سلام
خیلی ممنون

abay
17-09-2006, 14:29
سلام
علوم رياضي شريف
سلام
منم خدمت شما تبریک میگم اگه میشه رتبه ای که بدست آوردی ودرصدهای دروستون هم را بفرمایید

abay
18-09-2006, 21:26
با سلام

فرض کنید


[ برای مشاهده لینک ، لطفا با نام کاربری خود وارد شوید یا ثبت نام کنید ]

اگر برای هر x حقیقی داشته باشیم:


[ برای مشاهده لینک ، لطفا با نام کاربری خود وارد شوید یا ثبت نام کنید ]

ثابت کنید:


[ برای مشاهده لینک ، لطفا با نام کاربری خود وارد شوید یا ثبت نام کنید ]

موفق باشید.

ارسال متن: شنبه 25 شهریور 1385
سلام
آقای مفیدی در مورد حل سوال یه کم راهنمایی بفرمایید یه ذره سخت بنظر میرسه

mofidy1
19-09-2006, 14:44
سلام
آقای مفیدی در مورد حل سوال یه کم راهنمایی بفرمایید یه ذره سخت بنظر میرسه

با سلام

به مشتق f در نقطه صفر توجه کنید.

موفق باشید.

ali1234
21-09-2006, 09:45
سلام
علوم رياضي شريف
سلام آقاي پوران من هم ورود شما را به دانشگاه تبريك عرض ميكنم اميدوارم هميشه موفق وپيروز باشيد

ali_hp
21-09-2006, 10:59
سلام
از لطف همه دوستان ممنونم.

mofidy1
21-09-2006, 23:04
با سلام

خدمت دوستان عزیز عرض کنم که به لطف خدا فهرست مطالب اتاق ریاضیات ([ برای مشاهده لینک ، لطفا با نام کاربری خود وارد شوید یا ثبت نام کنید ]) تکمیل شد. سعی کردم به طور موضوعی اکثریت مطالب اتاق را طبقه بندی کنم تا مطالعه و جستجوی آنها آسانتر باشد. اگر نظری درباره این فهرست دارید لطفاً بیان فرمایید.

متشکرم.

mofidy1
21-09-2006, 23:05
با سلام

فرض کنید


[ برای مشاهده لینک ، لطفا با نام کاربری خود وارد شوید یا ثبت نام کنید ]

اگر برای هر x حقیقی داشته باشیم:


[ برای مشاهده لینک ، لطفا با نام کاربری خود وارد شوید یا ثبت نام کنید ]

ثابت کنید:


[ برای مشاهده لینک ، لطفا با نام کاربری خود وارد شوید یا ثبت نام کنید ]

موفق باشید.

ارسال متن: شنبه 25 شهریور 1385

با سلام

با توجه به فرض می توان نوشت:


[ برای مشاهده لینک ، لطفا با نام کاربری خود وارد شوید یا ثبت نام کنید ]

که حل مساله را کامل می کند. ( البته این مساله را با استقراء روی تعداد جملات تابع f نیز می توان حل کرد. برای دیدن حالت کلی تری از این مساله که با استقراء ثابت شده است به اینجا ([ برای مشاهده لینک ، لطفا با نام کاربری خود وارد شوید یا ثبت نام کنید ]) مراجعه فرمایید.)

(کتاب حل مساله از طریق مساله، ترجمه علی ساوجی)

موفق باشید.

ارسال متن: جمعه 31 شهریور 1385

mofidy1
21-09-2006, 23:07
با سلام

دوستان عزیز دانش آموز و دانشجو، آغاز سال تحصیلی جدید را خدمتتان تبریک گفته، آرزوی موفقیت روزافزون برایتان دارم. حدود 5 ماه از آغاز فعالیت این اتاق می گذرد و به لطف خدا با اقبال خوبی مواجه بوده است (این مطلب را می توان از تعداد پست های این اتاق و نیز تعداد بازدید های آن در مقایسه با موضوعات دیگر مطرح شده در p30world دریافت. تا این لحظه تعداد پستها 342 و تعداد بازدید ها 10277 است.) با این همه اتاق ریاضیات نقایص زیادی دارد که باید به کمک شما حل شود. برای تکمیل مطالب این اتاق در سال تحصیلی جدید چند پیشنهاد زیر را عنوان می کنم:

1- افزودن قسمت جدیدی به نام «مقالات کوتاه ریاضی» که در آن به نکات جالب و آموزنده درسی در سطح دبیرستان که معمولا در کلاس تدریس نمی شوند، اشاره شود تا ضمن بالا بردن سطح علمی دانش آموزان کمکی برای کنکور آنها باشد.

2- افتتاح بخش «تستهای ریاضی کنکور» و حل و بحث درباره آنها

3- افتتاح بخش «نمونه سوالات امتحانی» که دانش آموزان و دانشجویان را در دسترسی به سوالات امتحانی ریاضی کمک کند.

4- آموزش نرم افزار MAPLE که یکی از قویترین و فراگیر ترین نرم افزار های ریاضی است.

با توجه به اینکه این کارها را تک نفره نمی توان انجام داد از تمام بازدید کنندگان محترم تقاضای همکاری داریم. لطفا بفرمایید در کدام قسمت می توانید کمک کنید. در ضمن اگر پیشنهاد دیگری نیز دارید بیان فرمایید.

متشکرم.

mathcom
22-09-2006, 13:59
با سلام
آدرس لبخند رياضي به [ برای مشاهده لینک ، لطفا با نام کاربری خود وارد شوید یا ثبت نام کنید ] تغيير يافت. من زياد اهل فروم نيستم. مدير لطف كنه خودش يه كارايي براي تغيير بكنه.
خدانگهدار

abay
22-09-2006, 14:37
با سلام

با توجه به فرض می توان نوشت:


[ برای مشاهده لینک ، لطفا با نام کاربری خود وارد شوید یا ثبت نام کنید ]

که حل مساله را کامل می کند. ( البته این مساله را با استقراء روی تعداد جملات تابع f نیز می توان حل کرد.)

(کتاب حل مساله از طریق مساله، ترجمه علی ساوجی)

موفق باشید.

ارسال متن: جمعه 31 شهریور 1385
با سلام
آقای مفیدی حل مساله واقعا"خیلی جالب بود اگه میشه اون روش دیگه ای که گفتید اونو رو هم بنویسید

mofidy1
22-09-2006, 15:06
با سلام
آقای مفیدی حل مساله واقعا"خیلی جالب بود اگه میشه اون روش دیگه ای که گفتید اونو رو هم بنویسید

با سلام

دوست عزیز راهنمایی از بنده، حل با شما. این روش را خودتان به کار بگیرید و بعد از حل ما را هم از چگونگی آن با خبر کنید. اجازه دهید همه کارها را بنده انجام ندهم.

موفق باشید.

abay
22-09-2006, 20:47
با سلام

دوست عزیز راهنمایی از بنده، حل با شما. این روش را خودتان به کار بگیرید و بعد از حل ما را هم از چگونگی آن با خبر کنید. اجازه دهید همه کارها را بنده انجام ندهم.

موفق باشید.
سلام
اقای مفیدی در مورد این مسئله که قبلا" راهنمایی کرده بودید ((به مشتق f در نقطه صفر توجه کنید.))که میشه گفت جواب مسئله رو تقریبا" گفته بودید من نتونستم حل کنم چه برسه به اینکه الان فقط گفته اید با استقرا حل کنید من زیاد با استقرا میونه ی خوبی ندارم

mir@
23-09-2006, 13:17
سلام جناب مفیدی
خسته نباشید
شاید جای طرح این سوال در این تاپیک نباشه که در این صورت پوزش من رو بپذیرید.

اگر ممکنه در مورد نرم افزار Latex دارای قابلیت فارسی و محل دسترسی و نحوه نصب و آخرین ورژن و ... توضیحاتی بفرمایید.

من شنیده ام که Latex فارسی خیلی قدیمیه و بسیاری از Package ها رو ساپورت نمی کنه.

با سپاس فراوان
ارادتمند

eh_mn
23-09-2006, 13:18
با سلام
مطلوبست محاسبه حد زیر

[ برای مشاهده لینک ، لطفا با نام کاربری خود وارد شوید یا ثبت نام کنید ]

Esikhoob
23-09-2006, 19:37
با سلام خدمت دوستان
یکی از بستگان نزدیک من معلم ریاضیات جدید است ، از من خواسته که برای او راههای جدیدی (به کمک کامپیوتر) برای آموزش ریاضیات گسسته ، پیدا کنم .
منم فکر کردم چیزهایی مثل java-applet و animation که آنها را برای مباحثی مثل فیزیک مفید دیدم ، شاید در مورد ریاضیات گسسته پیدا شوند ، از دوستانی که دستی بر آتش دارند تقاضا دارم در مورد مباحث زیر:
'گراف - تئوری اعداد (همنهشتی و ..) - ترکیبیات (اصل شمول و عدم شمول) - احتمالات
هر کدام را که می توانید ، یک روش خوب ، راه حل خوب ، فیلم خوب ، CD آموزشی خوب ، خلاصه یک چیز ابتکاری ( بیشتر منظورم فیلم و عکس و چیزهای ملموس) هر چه قدر هم کوچک باشد ، به ما یاد بدهید .

mofidy1
23-09-2006, 20:02
سلام جناب مفیدی
خسته نباشید
شاید جای طرح این سوال در این تاپیک نباشه که در این صورت پوزش من رو بپذیرید.

اگر ممکنه در مورد نرم افزار Latex دارای قابلیت فارسی و محل دسترسی و نحوه نصب و آخرین ورژن و ... توضیحاتی بفرمایید.

من شنیده ام که Latex فارسی خیلی قدیمیه و بسیاری از Package ها رو ساپورت نمی کنه.

با سپاس فراوان
ارادتمند

با سلام

دوست عزیز به تازگی نسخه جدیدی از فارسی تک-ftex- که همان نرم افزار Latex دارای قابلیت فارسی است به وسیله دانشگاه صنعتی شریف طراحی شده است که در آن مشکلات قبلی تا حدی زیادی مرتفع شده است. برای اطلاعات بیشتر و نیز دانلود آن به آدرس زیر مراجعه فرمایید.

[ برای مشاهده لینک ، لطفا با نام کاربری خود وارد شوید یا ثبت نام کنید ]

موفق باشید.

mofidy1
23-09-2006, 20:10
با سلام

یک مثلث قائم الزاویه با اضلاع صحیح در نظر بگیرید. رئوس آن را به مرکز ثقل مثلث وصل کنید تا سه مثلث کوچکتر به دست آید. ثابت کنید مساحتهای این سه مثلث، اعداد صحیح زوج هستند.

موفق باشید.

ارسال متن: شنبه 1 مهر 1385

ali_hp
24-09-2006, 12:21
با سلام

یک مثلث قائم الزاویه با اضلاع صحیح در نظر بگیرید. رئوس آن را به مرکز ثقل مثلث وصل کنید تا سه مثلث کوچکتر به دست آید. ثابت کنید مساحتهای این سه مثلث، اعداد صحیح زوج هستند.

موفق باشید.

ارسال متن: شنبه 1 مهر 1385
سلام
به سادگي مي توان ديد كه مساحت سه مثلث كوچكتر برابر باهم و برابر يك سوم مساحت مثلث بزرگتر است پس كافي است ثابت كنيم مساحت مثلث بزرگ مضرب شش است.وتر مثلث قايم الزاويه راc ودوضلع ديگر را a,b بگيريد
طبق رابطه فيثاغورث داريم:
a^2+b^2=c^2
با توجه به اينكه مساحت مثلث برابر با ab/2 است بايد ثابت كنيم ab هم مضرب چهار است هم مضرب سه
لم:
الف) اگر x عددي صحيح فرد باشد x^2 به پيمانه هشت برابر يك است.واگر مضرب4 باشد x^2 به پيمانه هشت برابر 0 و اگر مضرب 2 باشد و مضرب چهار نباشد x^2 به پيمانه هشت برابر چهار خواحد بود.
ب)اگر x عددي صحيح باشد x^2 به پيمانه سه برابر صفر يا يك است.(اگر x مضرب 3 نباشد x^2 به پيمانه 3 برابر يك است.
اگر a مضرب 3 نباشدو bمضرب 3 نباشد با توجه به رابطه فيثاغورث و لم ب تساوي زير را به پيمانه 3 داريم كه تناقض است.
1+1=0,1
پس حداقل يكي از a,b مضرب 3 و در نتيجه حاصلضرب ab مضرب 3 است.
اگر a مضرب چهار نباشد و b مضرب چهار نباشد و چنين نباشد كه هر دو a,b زوج باشند با توجه به رابطه فيثاغورث و لم الف به يكي از تساويهاي زير مي رسيم
(در حالتي كه a,b فرد باشند): 1+1=0,1,4 كه غير ممكن است.
(درحالتي كه a فرد و b زوج باشدو برعكس) 1+4=0,1,4 كه غير ممكن است .
پس حداقل يكي از a,b مضرب 4 هستند و ياهر دو a,b زوج هستند كه در هر دو حالت ab مضرب چهار است.

mir@
25-09-2006, 11:13
سلام آقای مفیدی

من به تازگی با امکانات فوق العاده LATEX آشنا شده ام. لطفا بفرمایید برای آغاز کار کدام یک از دو ادیتور زیر بهتر است:
WinEdt یا Texniccenter.
در ضمن اگر کرک WinEdt v5.5 را هم در اختیار دارید متشکر خواهم شد.
من اخیراً شنیده ام که ورژن جدیدی از فارسی تک که LATEX 2e را ساپورت می کند توسط سازمان مدیریت خریداری شده است. آیا این گفته صحت دارد؟؟
با سپاس فراوان
ارادتمند

ali1234
27-09-2006, 09:01
با سلام
آقاي مفيدي در مورد سوال هفته قبل كه فرموديد از طريق استقرا هم مي توان آن را حل كرد. من الان دو روز است كه سعي ميكنم از اين طريق آن را حل كنم ولي به نتيجه اي نرسيدم مي خواستم لطف كنيد وخودتون حلش رو اينجا بگذاريد.
با تشكر فراوان.

ali_hp
27-09-2006, 11:34
با سلام
مطلوبست محاسبه حد زیر

[ برای مشاهده لینک ، لطفا با نام کاربری خود وارد شوید یا ثبت نام کنید ]
سلام
فرض كنيد مقدار حد t باشد(براي اثبات همگرايي دنباله مي توان ثابت كرد كه نزولي است ومثبت است پس همگراست) پس داريم :
[ برای مشاهده لینک ، لطفا با نام کاربری خود وارد شوید یا ثبت نام کنید ]
قرار دهيد:
[ برای مشاهده لینک ، لطفا با نام کاربری خود وارد شوید یا ثبت نام کنید ]
به سادگي مي توان ديد كه رابطه زير برقرار است:
[ برای مشاهده لینک ، لطفا با نام کاربری خود وارد شوید یا ثبت نام کنید ]
پس داريم:
[ برای مشاهده لینک ، لطفا با نام کاربری خود وارد شوید یا ثبت نام کنید ]
كه e عدد نپر است.

mofidy1
27-09-2006, 18:17
با سلام
مطلوبست محاسبه حد زیر

[ برای مشاهده لینک ، لطفا با نام کاربری خود وارد شوید یا ثبت نام کنید ]

با سلام

آقایان منبتی و حسین پوران از مساله و راه حل زیبایتان لذت بردم. با اجازه شما برای حل این مساله روش دیگری پیشنهاد می کنم که البته کمی تخصصی اما کوتاه تر است:

همانطور که آقای حسین پوران اشاره کردند این دنباله همگراست. حال بنابر فرمول استرلینگ داریم:


[ برای مشاهده لینک ، لطفا با نام کاربری خود وارد شوید یا ثبت نام کنید ]

حال چون حد مخرج 1 است، جواب حد به دست می آید.


طاعات و عبادات دوستان خوبم در این ماه پربرکت مقبول حق.
موفق باشید.

ali1234
28-09-2006, 09:52
با سلام
مطلوبست محاسبه حد زیر

[ برای مشاهده لینک ، لطفا با نام کاربری خود وارد شوید یا ثبت نام کنید ]
با سلام
مثل اينكه كمي دير رسيديم. [ برای مشاهده لینک ، لطفا با نام کاربری خود وارد شوید یا ثبت نام کنید ]

vgssbs66
28-09-2006, 11:41
سلام آقای مفیدی

من به تازگی با امکانات فوق العاده LATEX آشنا شده ام. لطفا بفرمایید برای آغاز کار کدام یک از دو ادیتور زیر بهتر است:
WinEdt یا Texniccenter.
در ضمن اگر کرک WinEdt v5.5 را هم در اختیار دارید متشکر خواهم شد.
من اخیراً شنیده ام که ورژن جدیدی از فارسی تک که LATEX 2e را ساپورت می کند توسط سازمان مدیریت خریداری شده است. آیا این گفته صحت دارد؟؟
با سپاس فراوان
ارادتمند

من يكي از گردانندگان وبلاگ فارسي تك ([ برای مشاهده لینک ، لطفا با نام کاربری خود وارد شوید یا ثبت نام کنید ]) هستم. ميشه بگين اين خبر نسخه ي جديد فارسي تك رو از كجا شنيدين. مي تونم از شما خواهش كنم با هم يك چت داشته باشيم؟

Balrog
29-09-2006, 23:52
كسى در مورد قاعده‌ى بخشپذيرى بر 4، 13، 17 و 19 اطلاعاتى داره؟

mofidy1
29-09-2006, 23:59
با سلام

یک مثلث قائم الزاویه با اضلاع صحیح در نظر بگیرید. رئوس آن را به مرکز ثقل مثلث وصل کنید تا سه مثلث کوچکتر به دست آید. ثابت کنید مساحتهای این سه مثلث، اعداد صحیح زوج هستند.

موفق باشید.

ارسال متن: شنبه 1 مهر 1385

با سلام

از دوست خوبم آقای حسین پوران که مساله این هفته را در پست 352 ([ برای مشاهده لینک ، لطفا با نام کاربری خود وارد شوید یا ثبت نام کنید ]) حل کردند، تشکر می کنم. بنده راه دیگری را مطرح می کنم که از سه تاییهای فیثاغورثی استفاده می کند.

به شکل زیر توجه فرمایید:


[ برای مشاهده لینک ، لطفا با نام کاربری خود وارد شوید یا ثبت نام کنید ]

همانطور که آقای حسین پوران اشاره کردند با استفاده از خاصیت نقطه همرسی میانه ها می توان ثا بت کرد که z=y=x
که y، x و z مساحتهای سه مثلث داخلی است و اینکه x ، یک سوم مساحت مثلث ABC است. حال با فرض اینکه زاویه C قائمه است و با توجه به سه تاییهای فیثاغورثی می توان رابطه های (1) و (2) را نوشت که m، n و k اعداد صحیح مثبت هستند.


[ برای مشاهده لینک ، لطفا با نام کاربری خود وارد شوید یا ثبت نام کنید ]

حال با کمی دقت می توان ثابت کرد که صورت آخرین کسر در فرمول (2) بر 2 و 3 بخشپذیر است که حل مساله را کامل می کند.

این مساله در المپیاد داخلی سال 1986 اسپانیا مطرح شده بود.

موفق باشید.

ارسال متن: جمعه 7 مهر 1385

eh_mn
30-09-2006, 05:20
با سلام
مطلوبست محاسبه حد زیر

[ برای مشاهده لینک ، لطفا با نام کاربری خود وارد شوید یا ثبت نام کنید ]

سلام

از همه دوستان بخاطر راه حل های خوبشان متشکرم. این هم راه حلی دیگر. ( که البته از من نیست)
فرض کنیم
[ برای مشاهده لینک ، لطفا با نام کاربری خود وارد شوید یا ثبت نام کنید ]
و
[ برای مشاهده لینک ، لطفا با نام کاربری خود وارد شوید یا ثبت نام کنید ] . در اینصورت
[ برای مشاهده لینک ، لطفا با نام کاربری خود وارد شوید یا ثبت نام کنید ]
در نتیجه
[ برای مشاهده لینک ، لطفا با نام کاربری خود وارد شوید یا ثبت نام کنید ]

موفق باشید

mofidy1
30-09-2006, 19:48
با سلام
آقاي مفيدي در مورد سوال هفته قبل كه فرموديد از طريق استقرا هم مي توان آن را حل كرد. من الان دو روز است كه سعي ميكنم از اين طريق آن را حل كنم ولي به نتيجه اي نرسيدم مي خواستم لطف كنيد وخودتون حلش رو اينجا بگذاريد.
با تشكر فراوان.

با سلام

دوست عزیز، حالت کلی تر این مساله (مساله هفته هفدهم) را (به همراه حل آن به وسیله استقرا) در لینک زیر مطالعه فرمایید. اگر سوالی داشتید مطرح فرمایید.

(منبع مساله: کتاب حل مساله از طریق مساله ، ترجمه علی ساوجی)

[ برای مشاهده لینک ، لطفا با نام کاربری خود وارد شوید یا ثبت نام کنید ]

موفق باشید.

mofidy1
30-09-2006, 20:08
با سلام

فرض کنید y، x و z سه عدد فرد طبیعی باشند. حداقل با دو روش متفاوت ثابت کنید:


[ برای مشاهده لینک ، لطفا با نام کاربری خود وارد شوید یا ثبت نام کنید ]

موفق باشید.

ارسال متن: شنبه 8 مهر 1385

ali1234
01-10-2006, 09:33
سلام آقاي مفيدي
در وبلاگ بسيار زيبا ومفيد شما لينكي وجود داشت با عنوان "راهكار هاي حل مسئله از آقاي جواد حاجي بابايي( [ برای مشاهده لینک ، لطفا با نام کاربری خود وارد شوید یا ثبت نام کنید ])" كه متاسفانه باز نشد مي خواستم كه اگه براتون ممكن است لينك رو اصلاح كنيد همچنين اگه خود شما تجربياتي در اين ضمينه داريدمانند توضيحات بسيار مفيد شما پيرامون روشهاي مطالعه رياضات. در اختيار ما بگزاريد.همچنيين از قرار دادن حل مسئله قبل به روش استقرائ بسيار ممنونم.
با تشكر.

ali_hp
01-10-2006, 13:01
كسى در مورد قاعده‌ى بخشپذيرى بر 4، 13، 17 و 19 اطلاعاتى داره؟
سلام
چهار:
يك: براي پيدا كردن باقيمانده تقسيم يك عدد بر چهار كافي است باقيمانده (دو برابر رقم دهگان + رقم يكان) ان عدد را بر چهار پيدا كنيم.
مثال:
[ برای مشاهده لینک ، لطفا با نام کاربری خود وارد شوید یا ثبت نام کنید ]
پس باقيمانده عدد اوليه بر چهار برابر يك است و بخش پذير نيست.
دو:براي سيزده:يك عدد سيزده بخش پذير است اگر وتنها اگر ( چهار برابر رقم يكان + عدد حاصل از حذف رقم يكان) برسيزده بخش پذير باشد.
مثال:
[ برای مشاهده لینک ، لطفا با نام کاربری خود وارد شوید یا ثبت نام کنید ]
با توجه به اينكه 45 بر 13 بخش پذير نيست عدد اوليه نيز بر 13 بخش پذير نيست
سه:براي هفده:يك عدد بر هفده بخش پذير است اگر وتنها اگر ( پنج برابر رقم يكان-عدد حاصل از حذف رقم يكان) برسيزده بخش پذير باشد.
مثال:
[ برای مشاهده لینک ، لطفا با نام کاربری خود وارد شوید یا ثبت نام کنید ]
چون منفي نه بر 17 بخش پذير نيست پس عدد اوليه بر17 بخش پذير نيست.
چهار:براي نوزده:يك عدد بر نوزده بخش پذير است اگر وتنها اگر (دو برابر رقم يكان + عدد حاصل از حذف رقم يكان) بر نوزده بخش پذير باشد.

mofidy1
01-10-2006, 23:02
سلام آقاي مفيدي
در وبلاگ بسيار زيبا ومفيد شما لينكي وجود داشت با عنوان "راهكار هاي حل مسئله از آقاي جواد حاجي بابايي( [ برای مشاهده لینک ، لطفا با نام کاربری خود وارد شوید یا ثبت نام کنید ])" كه متاسفانه باز نشد مي خواستم كه اگه براتون ممكن است لينك رو اصلاح كنيد همچنين اگه خود شما تجربياتي در اين زمينه داريدمانند توضيحات بسيار مفيد شما پيرامون روشهاي مطالعه رياضات. در اختيار ما بگزاريد.همچنيين از قرار دادن حل مسئله قبل به روش استقرائ بسيار ممنونم.
با تشكر.

با سلام

دوست عزیز بخش مقالات سایت تبیان دچار مشکل شده و احتمالا دسترسی به آنها فعلا امکان پذیر نیست. متاسفانه بنده نسخه ای از آن را هم در اختیار ندارم که خدمتتان تقدیم کنم. اگر مشکل حل شد خبرتان خواهم کرد. ضمنا یکی از بهترین منابع روشهای حل مساله کتاب «چگونه یک مساله را حل کنیم» تالیف «جرج پولیا» و ترجمه مرحوم «احمد آرام» است. به آنجا مراجعه فرمایید.

در اینجا جمله معروفی از جرج پولیا را نقل می کنم که در واقع خلاصه کتاب اوست: اگر می خواهید شنا یاد بگیرید وارد آب شوید و اگر می خواهید روشهای حل مساله را فرا بگیرید، مساله را حل کنید!

موفق باشید.

soleares
02-10-2006, 16:31
يكي لطف كنه سوال زير رو برام حل كنه :

[x]+[-x]+5x=4

متشكرم

mamo
02-10-2006, 17:18
سلام دوستان
مجموعه ای هست که تمام یا بخشی از تمرینات ریاضی 2 توی اون باشه ؟

ویا میتونین خودتون واسم حل کنین (فعلا صفحه 6 و 7 )

Marichka
02-10-2006, 18:42
سلام دوستان
مجموعه ای هست که تمام یا بخشی از تمرینات ریاضی 2 توی اون باشه ؟

ویا میتونین خودتون واسم حل کنین (فعلا صفحه 6 و 7 )

سلام دوست عزيز
سوالات خودتون رو در مورد مسايل رياضي در اين تاپيك مطرح كنيد تا سريعتر به پاسخ برسيد. در اگر مبحث خاصي مدنظرتون هست يا مسئله ي خاصي اگر به طور مشخص اينجا قرار بديد آقاي مفيدي عزيز و سايرين بهتر مي تونن كمكتون كنن.

در مورد حل المسايل هم كه الان موسسه هاي مختلف فكر مي كنم خيلي كتاب داشته باشن. فكر مي كنم آيندگان يا كتابهايي از اين دست.

ادغام شد :rolleye:
اميدوارم هميشه موفق باشيد :)

ali_hp
03-10-2006, 12:57
يكي لطف كنه سوال زير رو برام حل كنه :

[x]+[-x]+5x=4

متشكرم
سلام
اميدوارم صورت سوال همين باشد.
[ برای مشاهده لینک ، لطفا با نام کاربری خود وارد شوید یا ثبت نام کنید ]
مقدار عبارت
[ برای مشاهده لینک ، لطفا با نام کاربری خود وارد شوید یا ثبت نام کنید ]
براي x هاي صحيح برابر صفر و براي xهاي غير صحيح برابر منفي يك است.
حالا كافي است يك بار جوابهاي صحيح و يك بار جوابهاي غير صحيح را بدست اوريم.
اگر x عددي صحيح و صادق در رابطه مساله باشد داريم:
[ برای مشاهده لینک ، لطفا با نام کاربری خود وارد شوید یا ثبت نام کنید ]
كه جوابي قابل قبول نيست.
اگر x عددي غير صحيح در رابطه مساله باشد داريم:
[ برای مشاهده لینک ، لطفا با نام کاربری خود وارد شوید یا ثبت نام کنید ]
كه باز هم قابل قبول نيست.

ali_hp
03-10-2006, 13:20
با سلام

فرض کنید y، x و z سه عدد فرد طبیعی باشند. حداقل با دو روش متفاوت ثابت کنید:


[ برای مشاهده لینک ، لطفا با نام کاربری خود وارد شوید یا ثبت نام کنید ]

موفق باشید.

ارسال متن: شنبه 8 مهر 1385
سلام
برهان خلف:فرض كنيد تساوي زير براي سه عدد فرد x,y,z بر قرار باشد.
[ برای مشاهده لینک ، لطفا با نام کاربری خود وارد شوید یا ثبت نام کنید ]
اعداد صحيح a,b,c وجود دارند به طوري كه:
[ برای مشاهده لینک ، لطفا با نام کاربری خود وارد شوید یا ثبت نام کنید ]
با جايگذاري و ساده كردن به رابطه زير مي رسيم:
[ برای مشاهده لینک ، لطفا با نام کاربری خود وارد شوید یا ثبت نام کنید ] كه به وضوح غير ممكن است.

mamo
03-10-2006, 14:00
[ برای مشاهده لینک ، لطفا با نام کاربری خود وارد شوید یا ثبت نام کنید ]

mamo
03-10-2006, 21:31
7.5 ساعت بعد ..................

خب یکی به این جواب بده
...

mamo
03-10-2006, 21:35
[ برای مشاهده لینک ، لطفا با نام کاربری خود وارد شوید یا ثبت نام کنید ]

mofidy1
04-10-2006, 00:26
سلام
برهان خلف:فرض كنيد تساوي زير براي سه عدد فرد x,y,z بر قرار باشد.
[ برای مشاهده لینک ، لطفا با نام کاربری خود وارد شوید یا ثبت نام کنید ]
اعداد صحيح a,b,c وجود دارند به طوري كه:
[ برای مشاهده لینک ، لطفا با نام کاربری خود وارد شوید یا ثبت نام کنید ]
با جايگذاري و ساده كردن به رابطه زير مي رسيم:
[ برای مشاهده لینک ، لطفا با نام کاربری خود وارد شوید یا ثبت نام کنید ] كه به وضوح غير ممكن است.

با سلام

علی آقا ضمن تشکر از راه حلتان، سعی کنید روش دیگری نیز برای این مساله ارائه کنید.

ضمنا شما به تازگی از سایت [ برای مشاهده لینک ، لطفا با نام کاربری خود وارد شوید یا ثبت نام کنید ] برای آپلود فرمولهایتان استفاده می کنید. قبلا هم توضیح داده ام که امکان حذف آنها از روی server در این سایت وجود دارد همانطور که قبلا برای فرمولهای بنده این اتفاق افتاد و حسابی مرا به زحمت انداخت. در اولین فرصت به دنبال یک فضای شخصی مطمئن در اینترنت باشید تا خوانندگان از لطف شما همواره برخوردار باشند.

موفق باشید.

ali1234
04-10-2006, 09:59
[ برای مشاهده لینک ، لطفا با نام کاربری خود وارد شوید یا ثبت نام کنید ]
چون دو خط موازي هستند داريم:
b1+b2+c1+c2=180 چون b1=b2 و c1=c2 بنابرين b1+c1=90 بنابراين زاويه ديگر مثلثebc يعني زاويه e بايد 90 درجه باشد.

mamo
04-10-2006, 17:23
ممنون ali1234 جون دستت درد نکنه

faezeh7
05-10-2006, 11:15
يك سوال . يك عدد روز از سال را گرفته تعيين كند كه كدام روز از كدام ماه و كدام فصل است(فقط با محاسبات رياضي وبدون دستورات شرطي :blush:

mofidy1
06-10-2006, 00:09
با سلام

فرض کنید y، x و z سه عدد فرد طبیعی باشند. حداقل با دو روش متفاوت ثابت کنید:


[ برای مشاهده لینک ، لطفا با نام کاربری خود وارد شوید یا ثبت نام کنید ]

موفق باشید.

ارسال متن: شنبه 8 مهر 1385

با سلام

از آقای حسین پوران که در پست 371 ([ برای مشاهده لینک ، لطفا با نام کاربری خود وارد شوید یا ثبت نام کنید ]) به یکی از روشها اشاره کردند، تشکر می کنم. در لینک زیر سه روش برای حل این مساله زیبا ارائه شده است که روش اول همان روش آقای حسین پوران است.

[ برای مشاهده لینک ، لطفا با نام کاربری خود وارد شوید یا ثبت نام کنید ]

موفق باشید.

ارسال متن: حمعه 14 مهر 1385

Balrog
06-10-2006, 02:43
ali_hp! دستت درد نكنه!

Iron
06-10-2006, 04:02
[ برای مشاهده لینک ، لطفا با نام کاربری خود وارد شوید یا ثبت نام کنید ]

ali1234
06-10-2006, 09:39
با سلام
لطفا مقدار حد زير را بدست آوريد:
[ برای مشاهده لینک ، لطفا با نام کاربری خود وارد شوید یا ثبت نام کنید ]

ali1234
06-10-2006, 09:42
با سلام
چگونه مي شود پست هاي تكراري رو حذف كرد؟

mamo
06-10-2006, 17:53
مرسی Iron جان دستت درد نکنه

mofidy1
07-10-2006, 13:19
با سلام

فرض کنید Pn،...،P1 زیر مجموعه های دو عضوی متمایز از مجموعه {a1,...,an} باشند به طوریکه اگر اشتراک Pi و Pj ناتهی باشد، آنگاه مجموعه {ai , aj} یکی از P هاست. نشان دهید که هر a دقیقاً در یک جفت از P ها ظاهر می شود.

موفق باشید.

ارسال متن: شنبه 15 مهر 1385

attractive_girl
07-10-2006, 22:47
سلام درباره ي رياضي و يكي از فرمولاش و كاربردش(مثلا اتحاد اولر) ميخواستم مقاله اي برام بزارين ممنون ميشم

attractive_girl
07-10-2006, 22:50
سلام درباره ي رياضي و يكي از فرمولاش و كاربردش(مثلا اتحاد اولر) ميخواستم مقاله اي برام بزارين ممنون ميشم

eh_mn
07-10-2006, 23:42
با سلام
لطفا مقدار حد زير را بدست آوريد:
[ برای مشاهده لینک ، لطفا با نام کاربری خود وارد شوید یا ثبت نام کنید ]
سلام

[ برای مشاهده لینک ، لطفا با نام کاربری خود وارد شوید یا ثبت نام کنید ]

eh_mn
08-10-2006, 00:00
با سلام
اگر [ برای مشاهده لینک ، لطفا با نام کاربری خود وارد شوید یا ثبت نام کنید ] آنگاه مطلوبست مقدار [ برای مشاهده لینک ، لطفا با نام کاربری خود وارد شوید یا ثبت نام کنید ]

ali_hp
08-10-2006, 00:40
سلام

[ برای مشاهده لینک ، لطفا با نام کاربری خود وارد شوید یا ثبت نام کنید ]
سلام من فكر مي كنم حد وجود ندارد چون چون هر همسايگي به مركز صفر كه در نظر بگيريم x اي وجود دارد(در حقيقت بي نهايت x وجود دارد) كه سينوس يك x ام برابر صفر شود و در نتيجه عبارت بي معني شود.

Iron
08-10-2006, 01:22
با سلام
اگر [ برای مشاهده لینک ، لطفا با نام کاربری خود وارد شوید یا ثبت نام کنید ] آنگاه مطلوبست مقدار [ برای مشاهده لینک ، لطفا با نام کاربری خود وارد شوید یا ثبت نام کنید ]

[ برای مشاهده لینک ، لطفا با نام کاربری خود وارد شوید یا ثبت نام کنید ]

eh_mn
08-10-2006, 18:38
سلام من فكر مي كنم حد وجود ندارد چون چون هر همسايگي به مركز صفر كه در نظر بگيريم x اي وجود دارد(در حقيقت بي نهايت x وجود دارد) كه سينوس يك x ام برابر صفر شود و در نتيجه عبارت بي معني شود.
سلام
حق با شماست. در واقع در هر همسايگي از صفر يك مجموعه نامتناهي از اعداد به شكل [ برای مشاهده لینک ، لطفا با نام کاربری خود وارد شوید یا ثبت نام کنید ] وجود دارد. و تابع [ برای مشاهده لینک ، لطفا با نام کاربری خود وارد شوید یا ثبت نام کنید ] در اين مجموعه تعريف نشده است.

ali1234
13-10-2006, 10:16
سلام من فكر مي كنم حد وجود ندارد چون چون هر همسايگي به مركز صفر كه در نظر بگيريم x اي وجود دارد(در حقيقت بي نهايت x وجود دارد) كه سينوس يك x ام برابر صفر شود و در نتيجه عبارت بي معني شود.
سلام
از شما دو دوست عزيز بسيار متشكرم.

mofidy1
13-10-2006, 10:20
با سلام

فرض کنید Pn،...،P1 زیر مجموعه های دو عضوی متمایز از مجموعه {a1,...,an} باشند به طوریکه اگر اشتراک Pi و Pj ناتهی باشد، آنگاه مجموعه {ai , aj} یکی از P هاست. نشان دهید که هر a دقیقاً در یک جفت از P ها ظاهر می شود.

موفق باشید.

ارسال متن: شنبه 15 مهر 1385

با سلام

برای مشاهده حل این مساله به لینک زیر مراجعه فرمایید.

[ برای مشاهده لینک ، لطفا با نام کاربری خود وارد شوید یا ثبت نام کنید ]

(منبع: کتاب المپیادهای ریاضی بین المللی، ترجمه محمد قاسم وحیدی اصل)

موفق باشید.

ارسال متن: جمعه 21 مهر 1385

mofidy1
13-10-2006, 10:34
با سلام

دوستان عزیز دانش آموز و دانشجو، آغاز سال تحصیلی جدید را خدمتتان تبریک گفته، آرزوی موفقیت روزافزون برایتان دارم. حدود 5 ماه از آغاز فعالیت این اتاق می گذرد و به لطف خدا با اقبال خوبی مواجه بوده است (این مطلب را می توان از تعداد پست های این اتاق و نیز تعداد بازدید های آن در مقایسه با موضوعات دیگر مطرح شده در p30world دریافت. تا این لحظه تعداد پستها 342 و تعداد بازدید ها 10277 است.) با این همه اتاق ریاضیات نقایص زیادی دارد که باید به کمک شما حل شود. برای تکمیل مطالب این اتاق در سال تحصیلی جدید چند پیشنهاد زیر را عنوان می کنم:

1- افزودن قسمت جدیدی به نام «مقالات کوتاه ریاضی» که در آن به نکات جالب و آموزنده درسی در سطح دبیرستان که معمولا در کلاس تدریس نمی شوند، اشاره شود تا ضمن بالا بردن سطح علمی دانش آموزان کمکی برای کنکور آنها باشد.

2- افتتاح بخش «تستهای ریاضی کنکور» و حل و بحث درباره آنها

3- افتتاح بخش «نمونه سوالات امتحانی» که دانش آموزان و دانشجویان را در دسترسی به سوالات امتحانی ریاضی کمک کند.

4- آموزش نرم افزار MAPLE که یکی از قویترین و فراگیر ترین نرم افزار های ریاضی است.

با توجه به اینکه این کارها را تک نفره نمی توان انجام داد از تمام بازدید کنندگان محترم تقاضای همکاری داریم. لطفا بفرمایید در کدام قسمت می توانید کمک کنید. در ضمن اگر پیشنهاد دیگری نیز دارید بیان فرمایید.

متشکرم.

با سلام

دوستان عزیز ، چشم به راه همکاران جدید هستیم. درباره پیشنهادات بالا فکر کردید؟!

منتظر نظرات شما هستم.

موفق باشید.

eh_mn
13-10-2006, 15:24
[ برای مشاهده لینک ، لطفا با نام کاربری خود وارد شوید یا ثبت نام کنید ]
بخاطر راه حل خوبي كه ارائه داديد متشكرم.

mofidy1
14-10-2006, 07:29
با سلام

تابع f با شرایط زیر را در نظر بگیرید:


[ برای مشاهده لینک ، لطفا با نام کاربری خود وارد شوید یا ثبت نام کنید ]

ثابت کنید مقدار f در نقطه 1982 برابر است با 660 .

موفق باشید.

ارسال متن: شنبه 22 مهر 1385

mofidy1
15-10-2006, 13:44
با سلام

دوستان به لینک زیر مراجعه کنید. فلش جالبی است.

[ برای مشاهده لینک ، لطفا با نام کاربری خود وارد شوید یا ثبت نام کنید ]

موفق باشید.

mofidy1
15-10-2006, 13:46
با سلام

دوستانی که اطلاعاتی پیرامون تابع گاما دارند، به لینک زیر مراجعه کنند. در آن مطالبی پیرامون مشتق این تابع معروف آورده شده است.

[ برای مشاهده لینک ، لطفا با نام کاربری خود وارد شوید یا ثبت نام کنید ]

موفق باشید.

mofidy1
15-10-2006, 14:00
بسم الله الرحمن الرحیم

با توکل بر خدای بزرگ و تکیه بر الطاف بی انتهای او و با اجازه از دوستان عزیزم، بخش جدیدی را در اتاق ریاضیات با نام «مجموعه مسائل ریاضی» افتتاح می کنیم. به لطف خدا بخش «مساله هفته» مورد توجه بسیاری از دوستان و کاربران عزیز قرار گرفت و تجربه موفقی بود. هدف این حقیر از افتتاح بخش «مجموعه مسائل ریاضی» ارائه مجموعه ای جالب از مسائل زیبای ریاضی به صورت آنلاین در سطوح مختلف دبیرستان و دانشگاه، برای استفاده اساتید و دبیران محترم ریاضی، دانش آموزان و دانشجویان، مراکز پرورش استعداد های درخشان، المپیاد یها و ... است. این مجموعه را به دو قسمت دبیرستانی و دانشگاهی تقسیم می کنیم تا دوستان عزیز در پیدا کردن مسائل دلخواه خود سر درگم نشوند (هر چند که تقسیم بعضی از مسائل به دبیرستانی یا دانشگاهی بسیار مشکل است). از دوستان عزیز دعوت می کنم تا با حل این مسائل (با ذکر شماره آنها) در غنی کردن این مجموعه بکوشند. اگر مساله یا مساله های خوبی در اختیار دارید و آنها را مناسب این قسمت می دانید، آنها را به اطلاع این حقیر برسانید تا در صورت امکان در این مجموعه گنجانده شوند.

توضیحات بیشتر:

- هدف اولیه این بخش، مجموعه ای شامل هزار مساله (500 مساله دبیرستانی و 500 مساله دانشگاهی) است؛ انشاءالله.

- به دلایل خاصی، منابع این مسائل معرفی نمی شوند؛ غالب این مسائل از کتابهای معتبر حل مساله و نیز سایتهای اینترنتی مناسب انتخاب خواهند شد.

- مسائل این بخش، همگی مساله های سخت با راه حلهای پیچیده نیستند و بین آنها مسائل بسیار ساده نیز وجود دارد. انتخاب آنها بر اساس زیبایی و استحکام ایده های به کار رفته در آنهاست.

- سعی بر این است که انشاءالله مجموعه این مسائل به صورت pdf در اختیار بازدید کنندگان محترم قرار داده شود.

- دوستانی که در حل این مسائل بیش از دیگران تلاش کنند با نظر مدیران سایت، به عنوان کاربر فعال انجمن ریاضی شناخته خواهند شد.

- اگر تصاویر فرمولها دیده نمی شوند، صفحه را refresh کنید و اگر باز تصاویر ظاهر نشدند، مشکل را در همین اتاق به بنده اطلاع دهید.

و آخر دعوانا ان الحمدلله رب العالمین

23 مهر 1385 برابر با 21 ماه مبارک رمضان 1427


==================================================


قسمت A : مسائل دبیرستانی

A1.از تساوی زیر تابع f را بیابید:


[ برای مشاهده لینک ، لطفا با نام کاربری خود وارد شوید یا ثبت نام کنید ]
------------------------------------------------------------------------------------------------------------------------------
A2. دستگاه معادلات زیر را حل کنید:


[ برای مشاهده لینک ، لطفا با نام کاربری خود وارد شوید یا ثبت نام کنید ]
------------------------------------------------------------------------------------------------------------------------------
A3. معادلات زیر را حل کنید:


[ برای مشاهده لینک ، لطفا با نام کاربری خود وارد شوید یا ثبت نام کنید ]
------------------------------------------------------------------------------------------------------------------------------
A4. طول ضلعهای مثلثی قائم الزاویه، سه جمله متوالی یک تصاعد حسابی هستند. ثابت کنید که نسبت طول ضلعهای این مثلث برابر است با 3:4:5.
------------------------------------------------------------------------------------------------------------------------------
A5. فرض کنید 1...a=111 و 05...b=1000 که در a «یک» ها، m مرتبه و در b «صفر» ها، m-1 مرتبه تکرار شده اند. ثابت کنید ab+1 مربع کامل است و ریشه دومab+1 را به همان صورت که a و b نوشته شده اند، بنویسید.
------------------------------------------------------------------------------------------------------------------------------
A6. ثابت کنید برای هر عدد طبیعی n داریم:


[ برای مشاهده لینک ، لطفا با نام کاربری خود وارد شوید یا ثبت نام کنید ]
------------------------------------------------------------------------------------------------------------------------------

==================================================


قسمت B : مسائل دانشگاهی

B1. ثابت کنید برای هر عدد طبیعی n داریم:


[ برای مشاهده لینک ، لطفا با نام کاربری خود وارد شوید یا ثبت نام کنید ]
----------------------------------------------------------------------------------------------------------------------------------
B2. فرض کنید A و B دو ماتریس مربعی هم مرتبه با درایه های مختلط باشند. گزاره شرطی زیر را ثابت کنید:


[ برای مشاهده لینک ، لطفا با نام کاربری خود وارد شوید یا ثبت نام کنید ]
----------------------------------------------------------------------------------------------------------------------------------
B3. فرض کنید R حلقه ای با مشخصه صفر باشد (R لزوما تعویضپذیر نیست). فرض کنید f،e و g عناصر خودتوان R باشند به طوریکه مجموع آنها صفر است. ثابت کنید که باید هر سه صفر باشند.
----------------------------------------------------------------------------------------------------------------------------------

ali_hp
16-10-2006, 21:44
سلام
فرض كنيد مقدار حد t باشد(براي اثبات همگرايي دنباله مي توان ثابت كرد كه نزولي است ومثبت است پس همگراست) پس داريم :
[ برای مشاهده لینک ، لطفا با نام کاربری خود وارد شوید یا ثبت نام کنید ]
قرار دهيد:
[ برای مشاهده لینک ، لطفا با نام کاربری خود وارد شوید یا ثبت نام کنید ]
به سادگي مي توان ديد كه رابطه زير برقرار است:
[ برای مشاهده لینک ، لطفا با نام کاربری خود وارد شوید یا ثبت نام کنید ]
پس داريم:
[ برای مشاهده لینک ، لطفا با نام کاربری خود وارد شوید یا ثبت نام کنید ]
كه e عدد نپر است.
سلام
اين راه حل غلط است !
از همه دوستاني كه با دقت راه حل را خوانده اند و اشتباه ان را تذكر داده اند متشكرم.

eh_mn
17-10-2006, 18:27
سلام

1. فرض كنيد [ برای مشاهده لینک ، لطفا با نام کاربری خود وارد شوید یا ثبت نام کنید ] نشان دهيد

[ برای مشاهده لینک ، لطفا با نام کاربری خود وارد شوید یا ثبت نام کنید ]
كراندار است!

2. دنباله اي مانند a_n ارائه دهيد كه [ برای مشاهده لینک ، لطفا با نام کاربری خود وارد شوید یا ثبت نام کنید ] واگرا ولي [ برای مشاهده لینک ، لطفا با نام کاربری خود وارد شوید یا ثبت نام کنید ] همگرا باشد.

Iron
17-10-2006, 23:08
با سلام

تابع f با شرایط زیر را در نظر بگیرید:


[ برای مشاهده لینک ، لطفا با نام کاربری خود وارد شوید یا ثبت نام کنید ]

ثابت کنید مقدار f در نقطه 1982 برابر است با 660 .

موفق باشید.

ارسال متن: شنبه 22 مهر 1385
[ برای مشاهده لینک ، لطفا با نام کاربری خود وارد شوید یا ثبت نام کنید ]

در ضمن مسائل بخش مجموعه مسائل ریاضی برای من نشون داده نمي شن.

mofidy1
18-10-2006, 07:38
در ضمن مسائل بخش مجموعه مسائل ریاضی برای من نشون داده نمي شن.

لینک تصاویر تعویض شد. مراجعه فرمایید و نتیجه را به بنده اطلاع دهید.

موفق باشید.

Iron
18-10-2006, 15:36
لینک تصاویر تعویض شد. مراجعه فرمایید و نتیجه را به بنده اطلاع دهید.

موفق باشید.

دستت در نكنه. درست شد.

mofidy1
20-10-2006, 09:28
با سلام

دوستان عزیز اشکال استدلال زیر چیست؟


[ برای مشاهده لینک ، لطفا با نام کاربری خود وارد شوید یا ثبت نام کنید ]

موفق باشید.

mofidy1
20-10-2006, 09:29
با سلام

تابع f با شرایط زیر را در نظر بگیرید:


[ برای مشاهده لینک ، لطفا با نام کاربری خود وارد شوید یا ثبت نام کنید ]

ثابت کنید مقدار f در نقطه 1982 برابر است با 660 .

موفق باشید.

ارسال متن: شنبه 22 مهر 1385

با سلام

از عضو جدید اتاق ریاضیات Iron که مساله بالا را در پست 402 حل کردند، متشکرم. برای دیدن راه حل ایشان به لینک زیر مراجعه فرمایید:

[ برای مشاهده لینک ، لطفا با نام کاربری خود وارد شوید یا ثبت نام کنید ]

در ضمن توجه کنید که تابعی با شرایط بالا موجود است؛ به طور مثال قرار دهید:


[ برای مشاهده لینک ، لطفا با نام کاربری خود وارد شوید یا ثبت نام کنید ]
که [x] جزء صحیح x است.


دوستان عزیز به بخش جدید اتاق ریاضیات -مجموعه مسائل ریاضی- ([ برای مشاهده لینک ، لطفا با نام کاربری خود وارد شوید یا ثبت نام کنید ]) نیز مراجعه فرمایید.

موفق باشید.

ارسال متن: جمعه 28 مهر 1385

ali_hp
20-10-2006, 11:34
با سلام

دوستان عزیز اشکال استدلال زیر چیست؟


[ برای مشاهده لینک ، لطفا با نام کاربری خود وارد شوید یا ثبت نام کنید ]

موفق باشید.
سلام
تعریف رادیکال x چیست؟

ali_hp
21-10-2006, 14:14
در ضمن توجه کنید که تابعی با شرایط بالا موجود است؛ به طور مثال قرار دهید:


[ برای مشاهده لینک ، لطفا با نام کاربری خود وارد شوید یا ثبت نام کنید ]
که [x] جزء صحیح x است.


ايا اگر تابعي با شرايط بالا وجود نداشته باشد مشكل خاصي پيش مي ايد؟

mofidy1
21-10-2006, 16:18
ايا اگر تابعي با شرايط بالا وجود نداشته باشد مشكل خاصي پيش مي ايد؟

با سلام

نه، به هیچ عنوان؛ این مثال فقط به کنجکاوی احتمالی جواب می دهد.

علی آقا نمی دانم چقدر فرصت دارید، اما اگر امکانش برایتان هست، سری به قسمت تازه افتتاح شده (مجموعه مسائل ریاضی) بزنید و به غنای آن بیفزایید.

موفق باشید.

mofidy1
21-10-2006, 16:28
با سلام

فرض کنید:


[ برای مشاهده لینک ، لطفا با نام کاربری خود وارد شوید یا ثبت نام کنید ]

ثابت کنید:


[ برای مشاهده لینک ، لطفا با نام کاربری خود وارد شوید یا ثبت نام کنید ]

موفق باشید.

ارسال متن: شنبه 29 مهر 1385

Iron
21-10-2006, 19:54
با سلام

از عضو جدید اتاق ریاضیات Iron که مساله بالا را در پست 402 حل کردند، متشکرم. برای دیدن راه حل ایشان به لینک زیر مراجعه فرمایید:

[ برای مشاهده لینک ، لطفا با نام کاربری خود وارد شوید یا ثبت نام کنید ]



خواهش مي كنم. قابلي نداشت. يعني من جوابو از پست خودم پاك كنم؟

mofidy1
21-10-2006, 20:10
خواهش مي كنم. قابلي نداشت. يعني من جوابو از پست خودم پاك كنم؟

با سلام

نه نیازی به این کار نیست. قصد بنده ماندگاری تصویر شما در این اتاق بود. قبلا هم از دوستان خواسته بودیم که در صورت امکان تصاویر را در یک فضای شخصی آپلود کنند، زیرا هیچ اطمینانی به فضاهای عمومی نیست و ممکن است بعد از مدتی آنها را از روی server خودشان پاک کنند. در این صورت مطالعه پست ناقص هیچ فایده ای ندارد. قبلا این بلا سر بنده هم آمده بود.

لطفا همکاریتان را با این اتاق ادامه دهید.

موفق باشید.

LeBron
21-10-2006, 22:14
سلام من سوم دبيرستان درس ميخونم به يه سوال برخوردم كه احتمالا مال پيشه اگه كسي ميدونه لطفا جواب من رو بده خواهشا!! :blush: :biggrin:


|x-a|+|y-B|=k از نظر تابع بودن و اينكه چه شكلي ميشه و مينيموم ماكسيموم و ...
يكي ديگه :
|y-B| - |x-a| = K


< آخریش: :blush:
(x-a)^2+(y-B)^2=R^2

ali_hp
22-10-2006, 20:44
با سلام

فرض کنید:


[ برای مشاهده لینک ، لطفا با نام کاربری خود وارد شوید یا ثبت نام کنید ]

ثابت کنید:


[ برای مشاهده لینک ، لطفا با نام کاربری خود وارد شوید یا ثبت نام کنید ]

موفق باشید.

ارسال متن: شنبه 29 مهر 1385
سلام
قرار دهيد:
[ برای مشاهده لینک ، لطفا با نام کاربری خود وارد شوید یا ثبت نام کنید ]
[ برای مشاهده لینک ، لطفا با نام کاربری خود وارد شوید یا ثبت نام کنید ]
داريم:
[ برای مشاهده لینک ، لطفا با نام کاربری خود وارد شوید یا ثبت نام کنید ]
(براي بدست اوردن رابطه بالا كافي است مقدارS,P را در S- P قرار دهیم و مخرج مشترک بگیریم حالا با توجه به اینکه
عبارت S- P در حالت x=y برای هر z صفر است باید بتوان یک عامل x-y از ان بیرون کشید به همین ترتیب باید بتوان
x-z,z-y هم از ان بیرون کشید و به این ترتیب می توان S- P رابه حاصلضرب عبارتهای ساده تر تجزیه کرد)
پس:
[ برای مشاهده لینک ، لطفا با نام کاربری خود وارد شوید یا ثبت نام کنید ]
[ برای مشاهده لینک ، لطفا با نام کاربری خود وارد شوید یا ثبت نام کنید ]
بنابراین ثابت کرده ایم:
[ برای مشاهده لینک ، لطفا با نام کاربری خود وارد شوید یا ثبت نام کنید ]
که همان حکم مساله است.

ali_hp
23-10-2006, 00:42
با سلام

نه، به هیچ عنوان؛ این مثال فقط به کنجکاوی احتمالی جواب می دهد.

علی آقا نمی دانم چقدر فرصت دارید، اما اگر امکانش برایتان هست، سری به قسمت تازه افتتاح شده (مجموعه مسائل ریاضی) بزنید و به غنای آن بیفزایید.

موفق باشید.
سلام
1)من چطور مي توانم به غناي مجموعه مسايل رياضي بيافزايم؟(متوجه منظورتان نشدم)
2)تعريف راديكال x در پست 405 چیست؟

ali_hp
23-10-2006, 02:09
سلام من سوم دبيرستان درس ميخونم به يه سوال برخوردم كه احتمالا مال پيشه اگه كسي ميدونه لطفا جواب من رو بده خواهشا!! :blush: :biggrin:


|x-a|+|y-B|=k از نظر تابع بودن و اينكه چه شكلي ميشه و مينيموم ماكسيموم و ...
يكي ديگه :
|y-B| - |x-a| = K


< آخریش: :blush:
(x-a)^2+(y-B)^2=R^2
سلام
مجموعه (x,y) های صادق در رابطهlx-al+lx-bl=k,k>o یک مربع به مرکز (a,b) است .که طول قطرش 2k است ویک قطرش موازی محور xها وقطر دیگرش موازی محور yها است و مختصات رووس
ان عبارتند از (a+k,b),(a-k,b),(a,b+k),(a,b-k).به وضوح این نمودار نمودار یک تابع نیست.
x دربازه [a-k,a+k] تغییر می کند.وy هم در بازه [b-k,b+k] تغییر می کند.
(در حالت k=0 فقط x=a,y=b در رابطه صدق می کنند و نمودار ما تک نقطه (a,b) است که می تواند معرف نموداریک تابع نیزباشد.اگرk<0 هیچ x,y ای در رابطه صدق نمی کنند.)
مجموعه (x,y) های صادق در رابطه lx-al- ly-bl=k که k بزرگتر مساوی صفر است.نمودار این رابطه از دو زاویه نود درجه تشکیل شده است به طوری که نیمساز این زاویه ها موازی محور xها هستندراس یکی نقطه (a+k,b) است وراس دیگری (a-k,b) است. ) زاویه ها مثل > < قرار می گیرند.دقت کنید که اگر k=0 راس زاویه ها یکی میشود و به هم می چسبندواگر k>0 راس زاویه ها ازهم فاصله دارند.(فاصله ای برابر2k ))
پس برای x های بین a-k و a+k ایگرگی وجود ندارد.
پس x در محدوده R-(a-k,a+k)l تغییر می کند.y هم هر مقداری می تواند بگیرد.
معادله l (x-a)^2+(y-b)^2=R^2 نیز معادله دایره ای به شعاع R ومرکز (a,b) است.
X ان در محدوده [a-R,a+R] تغییر می کند.y ان هم در محدوده [b-R,b+r] تغییر می کند.

mofidy1
23-10-2006, 08:18
سلام
1)من چطور مي توانم به غناي مجموعه مسايل رياضي بيافزايم؟(متوجه منظورتان نشدم)
2)تعريف راديكال x در پست 405 چیست؟

با سلام

علی آقا در مقدمه بخش مجموعه مسائل کاملا توضیح داده ام. منظورم حل این مسائل و نیز ارائه مسائل خوب دیگر است.

در ضمن مشکل آن استدلال در همین جاست. اعداد مختلط (بر خلاف اعداد حقیقی مثبت) ریشه دوم منحصر به فرد ندارند. اگر x مثبت باشد رادیکال x عددی مثبت و منحصر به فرد مانند a است که در معادله a^2=x صدق می کند؛ اما به طور صوری می توان گفت که رادیکال 1- برابر است با دو عدد مختلط i و i-.

موفق باشید.

ozgor
23-10-2006, 23:31
درجمعی 100 نفره تعدادی وکیل و مهندس حضور دارند،اگر از هر دو نفر حد اقل 1 نفر وکیل باشد
در آن جمع چند وکیل و مهندس داریم؟

ozgor
23-10-2006, 23:56
دستگاه زیر را حل کنید

3 = x + y

x5 + y5 = 33
=========================
ثابت کنید برای هر مثلث با اضلاع a و b و c داریم

a 2 b (a – b ) + b 2 c ( b – c ) + c 2 a ( c- a ) > 0
=========================

[ برای مشاهده لینک ، لطفا با نام کاربری خود وارد شوید یا ثبت نام کنید ]

امکان دارد همه چند جمله ای های زیر دو ریشه حقیقی داشته باشند ?

P(x) = ax2 + bx + c

R(x) = bx2 + cx + a

Q(x) = cx2 + ax + b

ozgor
23-10-2006, 23:57
آیا یک چند جمله ای (f(x وجود دارد به صورتی که (x+1)f(x)= xf(x-1) باشد ؟

ozgor
24-10-2006, 00:00
دور یک دایره پنج عدد ۱ و چهار عدد ۰ به ترتیب دلخواه قرار دارند . بین هر دو عدد مساوی ۰ و بین هر دو عدد نا مساوی ۱ قرار می دهیم و اعداد اولیه را پاک می کنیم . آیا با تکرار این عمل ممکن است به ۹ رقم ۰ برسیم ؟
===============
[ برای مشاهده لینک ، لطفا با نام کاربری خود وارد شوید یا ثبت نام کنید ]
یک اسب روی خانه سیاه یک صفحه شطرنجی بی نهایت قرار داده شده است . این اسب بعد از n حرکت درست و حساب شده چه تعدادی مربع می تواند درست کند /

ozgor
24-10-2006, 00:01
فرض کنید a , b اعدای از R هستند و f(x) = a cos x + b cos 3x و داریم f(x) > 1 و جواب ندارد . ثابت کنيد
1 ≤ | b |

ozgor
24-10-2006, 00:03
حاصلضرب 3 عدد حقیقی مثبت برابر 1 می شود و مجموعشان بزرگتر از مجموع معکوسشان است . ثابت کنید که یکی از سه عدد بزرگتر از 1 می باشد
[ برای مشاهده لینک ، لطفا با نام کاربری خود وارد شوید یا ثبت نام کنید ]
یک دایره به شش قطاع تقسیم شده است . اعداد 0 , 0 , 0 , 1 , 0 , 1 به ترتیب در این قطاع ها نوشته شده اند . حال دو قطاع مجاور را انتخاب و به هر کدام یک واحد اضافه کنید . آیا با تکرار این عمل می توانید اعداد داخل ۶ قطاع را برابر کنید ؟
===========
تعدادی حروف b و a و e روی تخته نوشته شده است . می توانیم به جای دو حرف e یک حرف e و به جای دو حرف a یک حرف b و به جای دو حرف b یک حرف a و به جای یک حرف a و یک حرف b از یک حرف e و به جای یک حرف a و یک حرف e از یک حرف a و به جای یک حرف b و یک حرف e از یک حرف b استفاده کنیم . ثابت کنید حرفی که در نهایت باقی می ماند به ترتیب انجام اعمال بستگی ندارد .

mamo
24-10-2006, 12:20
سلام دوستان
عیدتون مبارک باد

کی میتونه به صورت آسان وراحت فصل دوم کتاب ریاضی دوم دبیرستان (فقط رابطه و تابع=مخصوصا تابع) رو به من یاد بده

eh_mn
25-10-2006, 02:16
سلام

1. فرض كنيد [ برای مشاهده لینک ، لطفا با نام کاربری خود وارد شوید یا ثبت نام کنید ] نشان دهيد

[ برای مشاهده لینک ، لطفا با نام کاربری خود وارد شوید یا ثبت نام کنید ]
كراندار است!

2. دنباله اي مانند a_n ارائه دهيد كه [ برای مشاهده لینک ، لطفا با نام کاربری خود وارد شوید یا ثبت نام کنید ] واگرا ولي [ برای مشاهده لینک ، لطفا با نام کاربری خود وارد شوید یا ثبت نام کنید ] همگرا باشد.
با سلام
خوشبختانه سوال اول بصورت زير حل شد

[ برای مشاهده لینک ، لطفا با نام کاربری خود وارد شوید یا ثبت نام کنید ]
لطفا براي حل سوال دوم راهنمايي كنيد.

متشكرم

mofidy1
25-10-2006, 11:38
با سلام

برادر خوبم، آقای منبتی، از اینکه دیر به سوالتان جواب می دهم معذرت می خواهم. بنده معمولا صبر می کنم که دیگران به سوال جواب دهند؛ اما ظاهرا بقیه هم مثل بنده سرشان شلوغ است. در هر صورت عذر می خواهم.

برای قسمت دوم سوالتان دنباله زیر را در نظر بگیرید (که آنرا در یکی از کتابها دیده ام):

[ برای مشاهده لینک ، لطفا با نام کاربری خود وارد شوید یا ثبت نام کنید ]

در ضمن بخشی از سوال شما، یکی از تستهای کارشناسی ارشد سال 1375 است.

موفق باشید.

mofidy1
27-10-2006, 13:55
با سلام

فرض کنید:


[ برای مشاهده لینک ، لطفا با نام کاربری خود وارد شوید یا ثبت نام کنید ]

ثابت کنید:


[ برای مشاهده لینک ، لطفا با نام کاربری خود وارد شوید یا ثبت نام کنید ]

موفق باشید.

ارسال متن: شنبه 29 مهر 1385

با سلام

از دوست عزیزم آقای حسین پوران که در پست 414 مساله را به روش زیبایی حل کردند، متشکرم. بنده راه حل دیگری براساس نامساوی هندسی-حسابی در نظر داشتم؛ اما روش ایشان زیباتر و کوتاه تر است. برای دیدن راه حل ایشان به لینک زیر مراجعه کنید:

[ برای مشاهده لینک ، لطفا با نام کاربری خود وارد شوید یا ثبت نام کنید ]

موفق باشید.

ارسال متن: جمعه 5 آبان 1385

Sir Masoud
28-10-2006, 04:49
دستگاه زیر را حل کنید

3 = x + y

x5 + y5 = 33




حل ندارد.
x + y =3
5x + 5y = 5(x + y)=15

ali_hp
28-10-2006, 18:34
حل ندارد.
x + y =3
5x + 5y = 5(x + y)=15
منظور از x5 ایکس به توان پنج است نه پنج ضربدر ایکس.

ali_hp
28-10-2006, 21:07
فرض کنید a , b اعدای از R هستند و f(x) = a cos x + b cos 3x و داریم f(x) > 1 و جواب ندارد . ثابت کنيد
1 ≤ | b |
سلام
اين مساله درست نيست.مثال نقض:a=b=1/2(هردو عدد a,b برابر یک دوم باشند)

mofidy1
28-10-2006, 22:10
با سلام

دستگاه معادلات مثلثاتی زیر را در نظر بگیرید:


[ برای مشاهده لینک ، لطفا با نام کاربری خود وارد شوید یا ثبت نام کنید ]

ثابت کنید:


[ برای مشاهده لینک ، لطفا با نام کاربری خود وارد شوید یا ثبت نام کنید ]

موفق باشید.

ارسال متن: شنبه 6 آبان 1385

eh_mn
28-10-2006, 22:19
با سلام

برای قسمت دوم سوالتان دنباله زیر را در نظر بگیرید (که آنرا در یکی از کتابها دیده ام):

[ برای مشاهده لینک ، لطفا با نام کاربری خود وارد شوید یا ثبت نام کنید ]

موفق باشید.
با سلام.
از راهنمايي شما متشكرم.

LeBron
29-10-2006, 19:48
سلام من درباره اثبات فرمول برد y=aSinx+bCosx+C یه سوال دارم من وقتی اثبات میکنم به فاصله بسته c منهای 2 در رادیکل ab (شرمنده نمیدونم اینجا چطوری به زبون ریاضی بنویسم) اجتماع با c به اضافه 2 در رادیکال ab میرسم میخوام بدونم فرمولش همینه یا من اشتباه کردم؟

ali1234
31-10-2006, 09:57
[ برای مشاهده لینک ، لطفا با نام کاربری خود وارد شوید یا ثبت نام کنید ]

m_honarmand_j
31-10-2006, 19:13
سلام . من قبلا سوالي مشابه سوال شما ديدم. لطفا سوالاتي كه مطمئن هستيد در جاي ديگري پيدا نمي شوند ويا جالب هستند رو مطرح كنيد . چون كسي كه اطلاعات كمي هم در مورد تركيبيات داشته باشد به راحتي مي تواند اين سوالي رو كه شما مطرح كرديد حل كنه. لطفا اگه سوالات جالبي در مورد تركيبيات داريد بزاريد.

mofidy1
31-10-2006, 19:29
سلام . من قبلا سوالي مشابه سوال شما ديدم. لطفا سوالاتي كه مطمئن هستيد در جاي ديگري پيدا نمي شوند ويا جالب هستند رو مطرح كنيد . چون كسي كه اطلاعات كمي هم در مورد تركيبيات داشته باشد به راحتي مي تواند اين سوالي رو كه شما مطرح كرديد حل كنه. لطفا اگه سوالات جالبي در مورد تركيبيات داريد بزاريد.

با سلام

دوست عزیز سعی بنده این است که مساله در حد امکان زیبا، استاندارد و متعادل باشد. سوالی که در زمینه ترکیبیات مطرح شد، سوال یکی از المپیادهای ریاضی است و به نظر بنده سوالی نیست كه با اطلاعات كم و کاملا مقدماتی در مورد تركيبيات بتوان آنرا حل کرد. در هر صورت اگر راه حلی ساده تر از این دارید با کمال میل حاضریم آنرا ببینیم. در ضمن اگر مسائل خوبی در ترکیبیات دارید در اینجا مطرح کنید تا با کمک شما و سایر دوستان آنها را حل کنیم.

موفق باشید.

dvm0day
01-11-2006, 10:27
سلام دوست عزيز دقيقا منظورم همان چيزي است كه شما نوشتيد.
از پاسخ شما خوشم آمد.من تازه عضو شدم و دانشجوي زياضي هستم.خيلي دوست دارم ازمطالب جديد شما استفاده كنم و باهم تبادل اطلاعات كنيكم. براي شروع گفتم بد نيست بدونيد روش ساده اي براي اثبات واگرايي سري هارمونيك به كمك نامساوي وجود داره.اگه دوست داشتين بدونين برام همينجا بنویسید.

mofidy1
01-11-2006, 19:53
با سلام

دوست عزیز خوش آمدید.

بی صبرانه منتظر روش اثبات واگرايي این سری هستیم. منتهی به فهرست مطالب انجمن ریاضی که آدرس آنرا ذیلا عرض می کنم، سری بزنید و مطالب آنرا درباره طریقه درست فرمول نویسی در اینترنت مطالعه فرمایید.

[ برای مشاهده لینک ، لطفا با نام کاربری خود وارد شوید یا ثبت نام کنید ]

موفق باشید.

mofidy1
03-11-2006, 08:27
با سلام

دستگاه معادلات مثلثاتی زیر را در نظر بگیرید:


[ برای مشاهده لینک ، لطفا با نام کاربری خود وارد شوید یا ثبت نام کنید ]

ثابت کنید:


[ برای مشاهده لینک ، لطفا با نام کاربری خود وارد شوید یا ثبت نام کنید ]

موفق باشید.

ارسال متن: شنبه 6 آبان 1385

با سلام

برای مطالعه راه حل مساله به لینک زیر مراجعه فرمایید:

[ برای مشاهده لینک ، لطفا با نام کاربری خود وارد شوید یا ثبت نام کنید ]

دوستان عزیز به بخش جدید اتاق ریاضیات -مجموعه مسائل ریاضی- ([ برای مشاهده لینک ، لطفا با نام کاربری خود وارد شوید یا ثبت نام کنید ]) نیز مراجعه فرمایید.

موفق باشید.

ارسال متن: جمعه 12 آبان 1385

mofidy1
04-11-2006, 06:57
با سلام

دوستان عزیز، یکی از کاربران جدید، سوال زیر را در انجمن فیزیک مطرح کرده اند. اگر می توانید به این سوال پاسخ دهید:

یه منحنی معروفی هست به نام منحنی فیزیک که یه مسئله روسی هست. یه کشاورزی هست که یه سگ داره و این سگ به فاصله b ازکشاورز در راستای محور x قرار دارد و کشاورز درمبدا مختصات هست و کشاورز در راستای مثبت محور y از مبدا مختصات با سرعت ثابت v حرکت میکند و سگ هم می خواهد با سرعت 2v به او برسد ما میدونیم که شتاب در اینجا صفر است ومسیر حرکت سگ به سمت کشاورز یک سهمی است .
من ازتون معادله y = f(x) سگ را می خواهم.

[ برای مشاهده لینک ، لطفا با نام کاربری خود وارد شوید یا ثبت نام کنید ]

mofidy1
04-11-2006, 12:32
با سلام

دنباله زیر را در نظر بگیرید:


[ برای مشاهده لینک ، لطفا با نام کاربری خود وارد شوید یا ثبت نام کنید ]

ثابت کنید:


[ برای مشاهده لینک ، لطفا با نام کاربری خود وارد شوید یا ثبت نام کنید ]

موفق باشید.

ارسال متن: شنبه 13 آبان 1385

ali_hp
06-11-2006, 10:03
سلام من درباره اثبات فرمول برد y=aSinx+bCosx+C یه سوال دارم من وقتی اثبات میکنم به فاصله بسته c منهای 2 در رادیکل ab (شرمنده نمیدونم اینجا چطوری به زبون ریاضی بنویسم) اجتماع با c به اضافه 2 در رادیکال ab میرسم میخوام بدونم فرمولش همینه یا من اشتباه کردم؟
سلام
شما اشتباه کرده اید جواب درست این است:
[ برای مشاهده لینک ، لطفا با نام کاربری خود وارد شوید یا ثبت نام کنید ]

ali_hp
06-11-2006, 13:18
با سلام

دوستان عزیز، یکی از کاربران جدید، سوال زیر را در انجمن فیزیک مطرح کرده اند. اگر می توانید به این سوال پاسخ دهید:

یه منحنی معروفی هست به نام منحنی فیزیک که یه مسئله روسی هست. یه کشاورزی هست که یه سگ داره و این سگ به فاصله b ازکشاورز در راستای محور x قرار دارد و کشاورز درمبدا مختصات هست و کشاورز در راستای مثبت محور y از مبدا مختصات با سرعت ثابت v حرکت میکند و سگ هم می خواهد با سرعت 2v به او برسد ما میدونیم که شتاب در اینجا صفر است ومسیر حرکت سگ به سمت کشاورز یک سهمی است .
من ازتون معادله y = f(x) سگ را می خواهم.

[ برای مشاهده لینک ، لطفا با نام کاربری خود وارد شوید یا ثبت نام کنید ]
سلام
اولا: در شكل مساله سرعت كشاورز دو برابر سرعت سگ است و در صورت مساله برعكس.كدام درست است؟
دوما:چگونه مي توان ثابت كرد مسير حركت سگ سهمي است؟

ali_hp
06-11-2006, 18:13
با سلام

دنباله زیر را در نظر بگیرید:


[ برای مشاهده لینک ، لطفا با نام کاربری خود وارد شوید یا ثبت نام کنید ]

ثابت کنید:


[ برای مشاهده لینک ، لطفا با نام کاربری خود وارد شوید یا ثبت نام کنید ]

موفق باشید.

ارسال متن: شنبه 13 آبان 1385
سلام
داریم:
[ برای مشاهده لینک ، لطفا با نام کاربری خود وارد شوید یا ثبت نام کنید ]
با توجه به رابطه بالا و بااستفاده از استقرا به راحتی می توان ثابت کرد:
[ برای مشاهده لینک ، لطفا با نام کاربری خود وارد شوید یا ثبت نام کنید ]
پس با توجه به مثبت بودن جملات دنباله
[ برای مشاهده لینک ، لطفا با نام کاربری خود وارد شوید یا ثبت نام کنید ]
(مقدار
[ برای مشاهده لینک ، لطفا با نام کاربری خود وارد شوید یا ثبت نام کنید ] وجود دارد چون جمله عمومی سری مثبت و سری از بالا کراندار است.در حقیقت می توان ثابت کرد که
[ برای مشاهده لینک ، لطفا با نام کاربری خود وارد شوید یا ثبت نام کنید ]

eh_mn
07-11-2006, 00:03
سلام و خسته نباشيد.
فرض كنيم [ برای مشاهده لینک ، لطفا با نام کاربری خود وارد شوید یا ثبت نام کنید ] بگونه اي باشد كه f(1)=1 و
[ برای مشاهده لینک ، لطفا با نام کاربری خود وارد شوید یا ثبت نام کنید ] . نشان دهيد f(n)=n !!!

ali1234
07-11-2006, 10:11
[ برای مشاهده لینک ، لطفا با نام کاربری خود وارد شوید یا ثبت نام کنید ]
با سلام
كسي جوابي براي اين مسئله نداشت ؟!

ali_hp
07-11-2006, 19:00
با سلام
كسي جوابي براي اين مسئله نداشت ؟!
سلام
قايده زنجيره اي:
[ برای مشاهده لینک ، لطفا با نام کاربری خود وارد شوید یا ثبت نام کنید ]
حل مساله:
[ برای مشاهده لینک ، لطفا با نام کاربری خود وارد شوید یا ثبت نام کنید ]

ali1234
08-11-2006, 08:45
سلام
قايده زنجيره اي:
[ برای مشاهده لینک ، لطفا با نام کاربری خود وارد شوید یا ثبت نام کنید ]
حل مساله:
[ برای مشاهده لینک ، لطفا با نام کاربری خود وارد شوید یا ثبت نام کنید ]
با سلام
علي جان دستت درد نكنه
خيلي ممنون

math_ir_man
08-11-2006, 11:36
سلام منم يك عضو جديدم.بايك سوال خوب.
0=0
0=(1-1)+(1-1)+(1-1)+(1-1)+(1-1)+...
0=1+(1-1) +(1-1) +(1-1) +(1-1) +
0=1 :rolleye:

m_honarmand_j
08-11-2006, 22:52
سلام . يك مسئله . گرافي 120 را’س دارد . يك چهارتايي بد به چهار را’ي از گراف مي گوييم كه دقيقا در بينشان يك يال وجود داشته باشد . ماكسيمم تعداد چهار تايي بد را در اين گراف بيابيد .

m_honarmand_j
09-11-2006, 18:44
سلام . يه سوال جالب . يك مدرسه كه 100 دانش آموز دارد كه در هر روز يك تيم پنج نفري به مسابقات شطرنج مي فرستد. حد اقل چند روز نياز است تا هر دونفر يك بار با هم به مسابقه بروند . :tongue:

m_honarmand_j
09-11-2006, 23:32
سلام . يك سوال ساده . 10 نفر دور يك ميز نشستند . هر نفر مي گويد كه نفر سمت چپ من يا راست گو است و يا دروغ گو . به چند طريق مي توان اين افراد را از لحاظ راسگو و دروغگو بودن نام گذاري كرد ؟ :tongue:
اين سوال خيلي آسوني است . دو سوالي كه قبلا مطرح كردم كمر شكن هستند . اگه تونستيد اونارو حل كنيد . به قول معروف سوالايي ان كه سوال لوله كن ها رو لوله مي كنن . البته خيلي هم سخت نيستند . از اونا سخت تر هم دارم . :laughing: :evil:

mofidy1
10-11-2006, 10:46
با سلام

دنباله زیر را در نظر بگیرید:


[ برای مشاهده لینک ، لطفا با نام کاربری خود وارد شوید یا ثبت نام کنید ]

ثابت کنید:


[ برای مشاهده لینک ، لطفا با نام کاربری خود وارد شوید یا ثبت نام کنید ]

موفق باشید.

ارسال متن: شنبه 13 آبان 1385

با سلام

از دوست خوبم آقای حسین پوران که در پست 444 مساله بالا را حل کردند متشکرم. برای مشاهده راه حل ایشان به لینک زیر مراجعه فرمایید.

[ برای مشاهده لینک ، لطفا با نام کاربری خود وارد شوید یا ثبت نام کنید ]

موفق باشید.

ارسال متن: جمعه 19 آبان 1385

mofidy1
11-11-2006, 18:37
با سلام

به شکل زیر توجه کنید. فرض کنید ABC مثلثی دلخواه با سه زاویه حاده باشد. سه ارتفاع AD و BE و CF را امتداد دهید تا دایره محیطی را به ترتیب در سه نقطه P و Q و R قطع کنند.اگر h طول بزرگترین ارتفاع و s طول کوچکترین پاره خط از بین پاره خطهای AP و BQ و CR باشد ثابت کنید عدد 4h-3s نامنفی است.


[ برای مشاهده لینک ، لطفا با نام کاربری خود وارد شوید یا ثبت نام کنید ]

موفق باشید.

ارسال متن: شنبه 20 آبان 1385

ali1234
15-11-2006, 10:21
با سلام خدمت دوستان عزيز
مي خواستم ببينم كسي لغت نامه تخصصي رياضي جايي سراغ دارد؟

najmeh_vahed
17-11-2006, 01:18
كي ميتونه ثابت كنه معادله سياله y^2 = x^2 + 2 جواب نداره
درضمن ميدونيم اگه اين معادله سياله جواب داشته باشه معادله همنهشتي اون يعني
(y^2 = x^2 (mod n به ازاي هر n بايد جواب داشته باشه

najmeh_vahed
17-11-2006, 09:53
ما در معادلات سياله فقط دنبال جوابهاي صحيح ميگرديم

mofidy1
17-11-2006, 23:02
با سلام

به شکل زیر توجه کنید. فرض کنید ABC مثلثی دلخواه با سه زاویه حاده باشد. سه ارتفاع AD و BE و CF را امتداد دهید تا دایره محیطی را به ترتیب در سه نقطه P و Q و R قطع کنند.اگر h طول بزرگترین ارتفاع و s طول کوچکترین پاره خط از بین پاره خطهای AP و BQ و CR باشد ثابت کنید عدد 4h-3s نامنفی است.


[ برای مشاهده لینک ، لطفا با نام کاربری خود وارد شوید یا ثبت نام کنید ]

موفق باشید.

ارسال متن: شنبه 20 آبان 1385

با سلام

می دانیم که قرینه نقطه H نسبت به هر یک از اضلاع روی دایره محیطی است. حال با توجه به خواص مقدماتی مثلث و نیز نامساوی هندسی - حسابی می توان نوشت:


[ برای مشاهده لینک ، لطفا با نام کاربری خود وارد شوید یا ثبت نام کنید ]

موفق باشید.

ارسال متن: جمعه 26 آبان 1385

mofidy1
18-11-2006, 13:17
با سلام

همه اعداد طبیعی مانند n را پیدا کنید که برای آنها اعداد طبیعی n_k ، ...، n_2 ، n_1 همگی بزرگتر از 3 وجود داشته باشند به طوریکه:


[ برای مشاهده لینک ، لطفا با نام کاربری خود وارد شوید یا ثبت نام کنید ]

موفق باشید.

ارسال متن: شنبه 27 آبان 1385

G A B R I E L
20-11-2006, 16:22
سلام دوستان ... به فرمایش جناب مفیدی این مسئله ها رو که مربوط به هندسه است رو اینجا مطرح میکنم ...

کسی میتونه حلشون کنه :

[ برای مشاهده لینک ، لطفا با نام کاربری خود وارد شوید یا ثبت نام کنید ]

[ برای مشاهده لینک ، لطفا با نام کاربری خود وارد شوید یا ثبت نام کنید ]

da_ya64
21-11-2006, 10:03
ارسال
هر کی میتونه این سوال را حل کنه
انتگرال ln(sinx) 0<x<pi/2 یعنی ایکس از صفر تا دوم است

najmeh_vahed
24-11-2006, 13:08
:biggrin: برای تنوع این سوال هم بد نیست:
n نفر در یک اتاق قرار دارند. به چند حالت این n نفر می توانند با هم دوست شوند.
( :) از دوستان خواهش میکنم اگر کسی نمونه این سوال را قبلا مشاهده کرده به من اطلاع دهد)




نگفتي تو گروه هاي چند نفري دوست شن؟ دو به دو؟

mofidy1
24-11-2006, 19:38
با سلام

همه اعداد طبیعی مانند n را پیدا کنید که برای آنها اعداد طبیعی n_k ، ...، n_2 ، n_1 همگی بزرگتر از 3 وجود داشته باشند به طوریکه:


[ برای مشاهده لینک ، لطفا با نام کاربری خود وارد شوید یا ثبت نام کنید ]

موفق باشید.

ارسال متن: شنبه 27 آبان 1385

با سلام

فرض کنید عدد n در شرایط مساله صدق کند. می توان دید که اگر عدد n+1 توانی از عدد 2 باشد و این توان کمتر از 10 باشد، تنها توان مورد قبول بنابر شرایط مساله عبارت است از 3؛ یعنی عدد 7 یکی از اعداد مورد نظر است. حال فرض کنید m ، بزرگتر یا مساوی 10 باشد و


[ برای مشاهده لینک ، لطفا با نام کاربری خود وارد شوید یا ثبت نام کنید ]

اگر عدد l حداقل 10 باشد آنگاه


[ برای مشاهده لینک ، لطفا با نام کاربری خود وارد شوید یا ثبت نام کنید ]

لذا بنابر استقراء


[ برای مشاهده لینک ، لطفا با نام کاربری خود وارد شوید یا ثبت نام کنید ]

بنابر این


[ برای مشاهده لینک ، لطفا با نام کاربری خود وارد شوید یا ثبت نام کنید ]

حال چون همه n_i ها فرد و بزرگتر از 3 هستند، لذا


[ برای مشاهده لینک ، لطفا با نام کاربری خود وارد شوید یا ثبت نام کنید ]

و در نتیجه


[ برای مشاهده لینک ، لطفا با نام کاربری خود وارد شوید یا ثبت نام کنید ]

که تناقض است. بنابر این تنها عدد قابل قبول، همان 7 است.

موفق باشید.

ارسال متن: جمعه 3 آذر 1385

G A B R I E L
25-11-2006, 00:02
سلام دوستان ... به فرمایش جناب مفیدی این مسئله ها رو که مربوط به هندسه است رو اینجا مطرح میکنم ...

کسی میتونه حلشون کنه :

[ برای مشاهده لینک ، لطفا با نام کاربری خود وارد شوید یا ثبت نام کنید ]

[ برای مشاهده لینک ، لطفا با نام کاربری خود وارد شوید یا ثبت نام کنید ]

آقای مفیدی شما گفتید که اینجا باید مطرح شه تا جواب بگیرهم ولی تا بحال کسی نبوده که جواب بده ...
:sad:

attractive_girl
26-11-2006, 19:54
سلام شرط اينكه 3عدد تشكيل مثلث بدهند چيست؟ :blink:
ممنون

mofidy1
26-11-2006, 23:45
با سلام

سه گروه از دانشمندان ریاضی از سه کشور مختلف در یک کنفرانس گرد آمده اند. می خواهیم جلسات سه نفری از این دانشمندان تشکیل دهیم به طوری که از هر گروه فقط یک نفر شرکت داشته باشد و هر دو نفر دقیقاً در یک جلسه با هم شرکت کرده باشند.

الف) اگر این کار امکان پذیر باشد نشان دهید تعداد افراد هر سه گروه مساویند.

ب) در حالتی که تعداد افراد هر گروه ، سه باشد نشان دهید این عمل امکان پذیر است.

موفق باشید.

ارسال متن: یکشنبه 5 آذر 1385

m_honarmand_j
26-11-2006, 23:52
سلام . يه سوال :
يك قالب پنير سه در سه وجود دارد . موشي از يك گوشه ي پنير شروع به خوردن مي كند . پس از خوردن هر قطعه به قطعه ي مجاور مي رود . دو قطعه مجاورند اگر يك وجه مشترك داشته باشند و پس از خوردن هر قطعه ديگر نمي توان به آنجا بر گشت . آيا مي تواند طوري اين پنير را بخورد كه آخرين قطعه اي كه مي خورد قطعه ي وسطي باشد ؟ :blush: :puke:

ali_hp
28-11-2006, 00:13
سلام شرط اينكه 3عدد تشكيل مثلث بدهند چيست؟ :blink:
ممنون
سلام
سه عدد مثبت تشکیل مثلث می دهند اگر وتنها اگر :"عدد بزرگتر از مجموع دو عدد دیگراکیدا کوچکتر باشد"

ali_hp
28-11-2006, 00:23
سلام . يه سوال :
يك قالب پنير سه در سه وجود دارد . موشي از يك گوشه ي پنير شروع به خوردن مي كند . پس از خوردن هر قطعه به قطعه ي مجاور مي رود . دو قطعه مجاورند اگر يك وجه مشترك داشته باشند و پس از خوردن هر قطعه ديگر نمي توان به آنجا بر گشت . آيا مي تواند طوري اين پنير را بخورد كه آخرين قطعه اي كه مي خورد قطعه ي وسطي باشد ؟ :blush: :puke:
سلام
سه در سه یا سه در سه در سه؟

ali_hp
28-11-2006, 00:57
سلام دوستان ... به فرمایش جناب مفیدی این مسئله ها رو که مربوط به هندسه است رو اینجا مطرح میکنم ...

کسی میتونه حلشون کنه :

[ برای مشاهده لینک ، لطفا با نام کاربری خود وارد شوید یا ثبت نام کنید ]

[ برای مشاهده لینک ، لطفا با نام کاربری خود وارد شوید یا ثبت نام کنید ]
سلام
شاید خیلی دیر باشه ولی...
در مورد اولی:
[ برای مشاهده لینک ، لطفا با نام کاربری خود وارد شوید یا ثبت نام کنید ]
که البته در شکل H از D به B نزدیکتر است که لزومی ندارد که چنین باشد.پس باید حالتی که Dاز H به B نز دیک تر است نیز بررسی شود در ان حالت جواب می شود
[ برای مشاهده لینک ، لطفا با نام کاربری خود وارد شوید یا ثبت نام کنید ]

دومی:
مثلث AEF را حول A ودر جهت حرکت عقربه ساعت 90 درجه بچرخانید. پس AF بر AC منطبق می شود و AE در امتداد BA قرار می گیرد و AH بر AM عمود می شود حال در مثلث EBF (دقت کنید که پس از دوران C,F بر هم منطبق شده اند.) خط AM وسط دو ضلع EBوBF را به هم وصل می کند پس AM موازی ضلع سوم است و از انجایی که AH بر AM عمود است پس AH برٍEF نیز عمود است.(زیرا EFوAM موازیند)

m_honarmand_j
28-11-2006, 18:35
سلام
n نقطه آبي و n نقطه قرمز در صفحه وجود دارد . مي خواهيم هر نقطه آبي را به يك نقطه قرمز با خط هاي صاف وصل كنيم به طوري كه اين خطوط همديگر را قطع نكنند . ثابت كنيد اين كار همواره امكان پذير است . :tongue:

m_honarmand_j
28-11-2006, 18:43
سلام
در داخل يك مثلث نقطه اي دل خواه در نظر گرفته و آن نقطه را به رئوس مثلث وصل مي كنيم . ثابت كنيد كه با سه پاره خط بوجود آمده تشكيل مثلث مي دهند . ( البته فكر كنم مثلث اوليه بايد منساوي الاضلاع باشد . ) :tongue:

m_honarmand_j
28-11-2006, 20:30
سلام
9 نقطه در فظاي سه بعدي با مختصات صحيح وجود دارد . ثابت كنيد دوتا از اين نقطه ها هستند كه مختصات نقطه ي وسطشان عددي صحيح است .

G A B R I E L
28-11-2006, 21:10
سلام
شاید خیلی دیر باشه ولی...
در مورد اولی:
[ برای مشاهده لینک ، لطفا با نام کاربری خود وارد شوید یا ثبت نام کنید ]
که البته در شکل H از D به B نزدیکتر است که لزومی ندارد که چنین باشد.پس باید حالتی که Dاز H به B نز دیک تر است نیز بررسی شود در ان حالت جواب می شود
[ برای مشاهده لینک ، لطفا با نام کاربری خود وارد شوید یا ثبت نام کنید ]

دومی:
مثلث AEF را حول A ودر جهت حرکت عقربه ساعت 90 درجه بچرخانید. پس AF بر AC منطبق می شود و AE در امتداد BA قرار می گیرد و AH بر AM عمود می شود حال در مثلث EBF (دقت کنید که پس از دوران C,F بر هم منطبق شده اند.) خط AM وسط دو ضلع EBوBF را به هم وصل می کند پس AM موازی ضلع سوم است و از انجایی که AH بر AM عمود است پس AH برٍEF نیز عمود است.(زیرا EFوAM موازیند)

خدا خیرت بده ...

ممنونم ...
;)

m_honarmand_j
30-11-2006, 15:58
سلام .
در مورد اون سوال كه موشه مي خاست پنيرو بخوره , پنير مكعب سه در سه در سه هست .
(بابا نميتونين جواب سوال و پيدا كنين گيراي الكي ندين ديگه . )

m_honarmand_j
30-11-2006, 16:00
دوستان اون سوال كه نقطه اي در مثلث مي ذاشتيم و به راس هاي مثلث وصل مي كرديم , مثلث متساوي الاضلاع است .

m_honarmand_j
30-11-2006, 16:06
سلام .
يك جمعي از نمايندگان كشور ها مي خواهند در دورديف صندلي روبهروي هم بشينند .(دوطرف ميز) . هر كس حد اكثر 3 دشمن دارد . ثابت كنيد مي توان اين افراد را در اين دو رديف صندلي نشاند به طوري كه هر كس حد اكثر يك دشمن در رديفي كه نشسته است داشته باشد .

m_honarmand_j
30-11-2006, 16:10
سلام .
تعدادي خانه در دور يك ميدان قرار دارند به طوري كه سقف هر خانه يا قرمز است يا سبز و تعداد همسايه هاي هر خانه عددي فرد است . از يك خانه شروع كرده و رنگ آن را به رنگي كه بيشتر همسايه هايش دارند در مي آوريم . اين كار را دور تا دور ميدان انجام مي دهيم . ثابت كنيد به مرحله اي مي رسيم كه ديگر رنگ خانه ها عوض نمي شود .

Ac.Milan
30-11-2006, 19:04
با عرض سلام خدمت دوستان عزیز:
کسی اینجا هست که بتونه تابع فی در بخش نظریه اعداد کتاب گسسته پیش دانشگاهی رو برای من اثبات کنه؟؟؟

m_honarmand_j
30-11-2006, 19:46
سلام بروبچ
در يك n ضلعي محدب تمام قطر هاي آن را رسم مي كنيم . اگر هيچ سه قطري همراس نباشند , n ضلعي به چند بخش تقسيم مي شود ؟

m_honarmand_j
30-11-2006, 19:58
سلام
در روزگاران قديم , از طرف فرمانرواي شهري اين اطلاعيه صادر شده است : ((مسلم شده است در شهر زناني وجود دارند كه به شوهران خود خيانت ميكنند, و مسلم شده است كه بر خيانت هر يك از اين زنان , همه ي مردم شهر به غير از شوهر ايشان اطلاع دارند . از مردان خواسته مي شود كه فقط از روي تعقل و بدون تفحص از ديگران درباره ي زنان خود بينديشند و هر مرد كه در يافت زنش به وي خيانت مي كند بامداد روز بعد آن زن را به قتل برساند .)) ثابت كنيد بلاخره روزي فرا ميرسد كه تمام زنان خيانت كار كشته شده باشند .

attractive_girl
01-12-2006, 08:24
سلام
سه عدد مثبت تشکیل مثلث می دهند اگر وتنها اگر :"عدد بزرگتر از مجموع دو عدد دیگراکیدا کوچکتر باشد"

مرسي دعاي دوستم پشت و پناهته

azad666
01-12-2006, 17:58
سلام
به جواب تشریحی تست های 45و46کنکور ارشد 84درس آنالیز1نیاز مبرم دارم لطفا کمک کنید0

mofidy1
01-12-2006, 20:11
با سلام

معرفی یک سایت:

اتحادیه انجمن های معمان ریاضی ایران [ برای مشاهده لینک ، لطفا با نام کاربری خود وارد شوید یا ثبت نام کنید ]

mofidy1
01-12-2006, 22:49
با سلام

دوستان عزیز به لینک زیر سری بزنید و نبوغ خارق العاده بعضی ها را در پاسخ به سوالات ریاضی ببینید!!

[ برای مشاهده لینک ، لطفا با نام کاربری خود وارد شوید یا ثبت نام کنید ]

موفق باشید.

mofidy1
01-12-2006, 22:52
با سلام

سه گروه از دانشمندان ریاضی از سه کشور مختلف در یک کنفرانس گرد آمده اند. می خواهیم جلسات سه نفری از این دانشمندان تشکیل دهیم به طوری که از هر گروه فقط یک نفر شرکت داشته باشد و هر دو نفر دقیقاً در یک جلسه با هم شرکت کرده باشند.

الف) اگر این کار امکان پذیر باشد نشان دهید تعداد افراد هر سه گروه مساویند.

ب) در حالتی که تعداد افراد هر گروه ، سه باشد نشان دهید این عمل امکان پذیر است.

موفق باشید.

ارسال متن: یکشنبه 5 آذر 1385

با سلام

فرض کنید دانشمندان مجموعه های


[ برای مشاهده لینک ، لطفا با نام کاربری خود وارد شوید یا ثبت نام کنید ]

باشند. برای هر یک از اعضای مجموعه ها نقطه ای روی صفحه متناظر می کنیم (شکل زیر برای حالت 3=m=n=p نشان داده شده است).


[ برای مشاهده لینک ، لطفا با نام کاربری خود وارد شوید یا ثبت نام کنید ]

اگر یک دانشمند مثلا a_i با یک دانشمند مثلا b_j در یک جلسه شرکت داشته باشد یک خط بین این دو رسم می کنیم. تمام نقاط A را به هر یک از نقاط B و C و همین طور هر یک از نقاط B را به هر یک از نقاط C وصل می کنیم. منظور مساله، افراز خطوط حاصل به مثلثهایی است که سه راس آنها یکی در A ، یکی در B و یکی در C است.

الف) مثلثهای مذکور از هر قسمت {A,B}، {B,C} و {A,C} دقیقاً یک خط در بر دارد؛ پس باید تعدادخطوط بین این قسمتها با هم مساوی باشند. در نتیجه mp=mn و np=mn و لذا m=n=p.

ب) مثلثها می توانند به صورت زیر باشند:


[ برای مشاهده لینک ، لطفا با نام کاربری خود وارد شوید یا ثبت نام کنید ]

جدول زیر بیانگر حل این حالت از مساله است(a_i و b_j با c_k که از جدول به دست می آید، یک جلسه تشکیل خواهند داد و c_k در ستون a_i و سطر b_j قرار دارد.)


[ برای مشاهده لینک ، لطفا با نام کاربری خود وارد شوید یا ثبت نام کنید ]

(توضیح بیشتر: در حالت کلی نیز کافیست از جدولی همانند جدول بالا استفاده کنیم. ماتریسی که در داخل جدول بالا مشاهده می کنید به ماتریس دوری معروف است.)

موفق باشید.

ارسال متن: جمعه 10 آذر 1385

mofidy1
02-12-2006, 23:53
با سلام

فرض کنید:


[ برای مشاهده لینک ، لطفا با نام کاربری خود وارد شوید یا ثبت نام کنید ]

ثابت کنید:


[ برای مشاهده لینک ، لطفا با نام کاربری خود وارد شوید یا ثبت نام کنید ]

موفق باشید.

ارسال متن: شنبه 11 آذر 1385

eh_mn
03-12-2006, 19:13
سلام
به جواب تشریحی تست های 45و46کنکور ارشد 84درس آنالیز1نیاز مبرم دارم لطفا کمک کنید0

با سلام
لطفا صورت سوال ها را بنويسيد.

eh_mn
03-12-2006, 19:31
با سلام.

فرض كنيم مجموعه اعداد طبيعي به تعدادي تصاعد عددي افراز شده است. ثابت كنيد قدر نسبت حداقل دو تا از اين تصاعدها با هم برابرند و همچنين مجموع معكوس هاي (ضربي) قدر نسبت ها برابر واحد است !!!

موفق باشيد

Arash_XL7710i_207
03-12-2006, 21:05
اگه ميشه تبديل عدد اعشاري متناوب مركب رو به كسرش براي من تو ضيح بديد.

m_honarmand_j
03-12-2006, 23:53
سلام
دنبالا اي به طول از 0 و 1 داريم . چند تا از اين دنباله ها داراي دقيقا تا 01 هستند ؟

ali1234
04-12-2006, 09:43
سلام
دنبالا اي به طول از 0 و 1 داريم . چند تا از اين دنباله ها داراي دقيقا تا 01 هستند ؟
سلام دوست عزيز
دنبالهاي به طول چند؟؟؟و چند تا 01؟؟؟

azad666
04-12-2006, 19:44
بروبه این آدرس
[ برای مشاهده لینک ، لطفا با نام کاربری خود وارد شوید یا ثبت نام کنید ]

eh_mn
04-12-2006, 22:14
با سلام
سوال 45) مقدار
[ برای مشاهده لینک ، لطفا با نام کاربری خود وارد شوید یا ثبت نام کنید ]
كدام است ؟
1) 2
2)5/2
3)75/2
4) 3

مي بينيم كه
[ برای مشاهده لینک ، لطفا با نام کاربری خود وارد شوید یا ثبت نام کنید ] .
براي محاسبه مقدار اين سري ابتدا مقدار
[ برای مشاهده لینک ، لطفا با نام کاربری خود وارد شوید یا ثبت نام کنید ]
را (براي 1>|x|) بدست مي آوريم و سپس بجاي x عدد 2/1 را قرار مي دهيم.كافيست به اين نكته توجه كنيد كه سري اخير با مشتق گيري (و تغيير انديس با تقسيم بر يا ضرب در x) از سري
[ برای مشاهده لینک ، لطفا با نام کاربری خود وارد شوید یا ثبت نام کنید ]
بدست مي آيد.
با اندكي محاسبه معلوم مي شود كه گزينه 1 صحيح است.

سوال 46) اگر
[ برای مشاهده لینک ، لطفا با نام کاربری خود وارد شوید یا ثبت نام کنید ]
حاصل [ برای مشاهده لینک ، لطفا با نام کاربری خود وارد شوید یا ثبت نام کنید ] كدام است ؟
1) صفر
2) 2
3) 4
4) بينهايت

اگر E مجموعه حدود زيردنباله اي دنباله an باشد آنگاه [ برای مشاهده لینک ، لطفا با نام کاربری خود وارد شوید یا ثبت نام کنید ] و [ برای مشاهده لینک ، لطفا با نام کاربری خود وارد شوید یا ثبت نام کنید ] .

بنابراين كافيست حدود زيردنباله اي را پيدا كنيم.
فكر كنم براي حل اين مسئله بايد بدين ترتيب عمل كنيم كه
فرض كنيم [ برای مشاهده لینک ، لطفا با نام کاربری خود وارد شوید یا ثبت نام کنید ] زيردنباله اي از دنباله مذكور باشد. سه حالت در نظر مي گيريم
1) فقط تعدادي متناهي از اعضاي مجموعه [ برای مشاهده لینک ، لطفا با نام کاربری خود وارد شوید یا ثبت نام کنید ] به شكل 2n هستند.
2) فقط تعداد متناهي از اعضاي مجموعه [ برای مشاهده لینک ، لطفا با نام کاربری خود وارد شوید یا ثبت نام کنید ] به شكل 2n+1 هستند.
3) تعداد نامتناهي از اعضاي مجموعه [ برای مشاهده لینک ، لطفا با نام کاربری خود وارد شوید یا ثبت نام کنید ] به شكل 2n و تعداد نامتناهي از اعضاي مجموعه [ برای مشاهده لینک ، لطفا با نام کاربری خود وارد شوید یا ثبت نام کنید ] به شكل 2n+1 هستند.
مي توان نشان داد كه دنباله [ برای مشاهده لینک ، لطفا با نام کاربری خود وارد شوید یا ثبت نام کنید ] در حالت 3 حد ندارد و حد آن در دوحالت 1 و 2 به ترتيب برابر 2- و 2 مي شود.
بنابراين گزينه 3 درست است.

azad666
05-12-2006, 06:55
با سلام
سوال 45) مقدار
[ برای مشاهده لینک ، لطفا با نام کاربری خود وارد شوید یا ثبت نام کنید ]
كدام است ؟
1) 2
2)5/2
3)75/2
4) 3

مي بينيم كه
[ برای مشاهده لینک ، لطفا با نام کاربری خود وارد شوید یا ثبت نام کنید ] .
براي محاسبه مقدار اين سري ابتدا مقدار
[ برای مشاهده لینک ، لطفا با نام کاربری خود وارد شوید یا ثبت نام کنید ]
را (براي 1>|x|) بدست مي آوريم و سپس بجاي x عدد 2/1 را قرار مي دهيم.كافيست به اين نكته توجه كنيد كه سري اخير با مشتق گيري (و تغيير انديس با تقسيم بر يا ضرب در x) از سري
[ برای مشاهده لینک ، لطفا با نام کاربری خود وارد شوید یا ثبت نام کنید ]
بدست مي آيد.
با اندكي محاسبه معلوم مي شود كه گزينه 1 صحيح است.

سوال 46) اگر
[ برای مشاهده لینک ، لطفا با نام کاربری خود وارد شوید یا ثبت نام کنید ]
حاصل [ برای مشاهده لینک ، لطفا با نام کاربری خود وارد شوید یا ثبت نام کنید ] كدام است ؟
1) صفر
2) 2
3) 4
4) بينهايت

اگر E مجموعه حدود زيردنباله اي دنباله an باشد آنگاه [ برای مشاهده لینک ، لطفا با نام کاربری خود وارد شوید یا ثبت نام کنید ] و [ برای مشاهده لینک ، لطفا با نام کاربری خود وارد شوید یا ثبت نام کنید ] .

بنابراين كافيست حدود زيردنباله اي را پيدا كنيم.
فكر كنم براي حل اين مسئله بايد بدين ترتيب عمل كنيم كه
فرض كنيم [ برای مشاهده لینک ، لطفا با نام کاربری خود وارد شوید یا ثبت نام کنید ] زيردنباله اي از دنباله مذكور باشد. سه حالت در نظر مي گيريم
1) فقط تعدادي متناهي از اعضاي مجموعه [ برای مشاهده لینک ، لطفا با نام کاربری خود وارد شوید یا ثبت نام کنید ] به شكل 2n هستند.
2) فقط تعداد متناهي از اعضاي مجموعه [ برای مشاهده لینک ، لطفا با نام کاربری خود وارد شوید یا ثبت نام کنید ] به شكل 2n+1 هستند.
3) تعداد نامتناهي از اعضاي مجموعه [ برای مشاهده لینک ، لطفا با نام کاربری خود وارد شوید یا ثبت نام کنید ] به شكل 2n و تعداد نامتناهي از اعضاي مجموعه [ برای مشاهده لینک ، لطفا با نام کاربری خود وارد شوید یا ثبت نام کنید ] به شكل 2n+1 هستند.
مي توان نشان داد كه دنباله [ برای مشاهده لینک ، لطفا با نام کاربری خود وارد شوید یا ثبت نام کنید ] در حالت 3 حد ندارد و حد آن در دوحالت 1 و 2 به ترتيب برابر 2- و 2 مي شود.
بنابراين گزينه 3 درست است. خدا خيرت بده خيلي كمك كردي

m_honarmand_j
07-12-2006, 00:22
سلام
از دوستان پوزش مي خوام . در سوالي كه تعداد دنباله ها رو خواسته بودم . منظور تعداد دنباله هايي به طول n كه دقيقا داراي m تا 01 هستند بود و در سوال m و n جامونده بودند .